You are on page 1of 102

VIBGYOR HIGH

SAMPLE PAPER
PHYSICS
Grade: IX Max. Marks: 80
Date: Time Allowed: 2 hours

INSTRUCTIONS:

 Answers to this paper must be written on the paper provided separately.


 You will not be allowed to write during the first 15 minutes.
 This time is to be spent in reading the question paper.
 The time given at the head of this paper is the time allowed for writing the
answers.
 The intended marks for the questions or parts of questions are given alongside
the questions.
 This question paper contains 5 printed pages.
____________________________________________________________________

SECTION I (40 Marks)


Attempt all questions from this Section

Question 1 [10]
(a) Define variable speed. Give a suitable example.
[2]
(b) The time period of the two simple pendulums are in the ratio
4:9. Compare their effective lengths.
[2]
(c) Give any 2 points of difference between longitudinal waves and
transverse waves. [2]
(d) The figure below shows the metallic bob suspended
through a string to the rigid support. Name the forces acting on
the bob. Copy the diagram and show the direction of forces
acting on the bob.

[2]

1
(e) Why do porters put on a turban when they carry heavy load on [2]
their head?

Question 2 [10]
(a) A bullet striking against a block of wood with a velocity of
360 m/s pierces a hole in it 12 cm deep. Calculate the
deceleration of the bullet. [2]
(b) State the action and reaction forces in the following example:
A man walking on the ground. [2]
(c) Name two factors on which time period of a simple pendulum
does not depend. [2]
(d) A man standing in front of a special mirror finds that his image
is having a very small head, a fat body and legs of normal size.
What are the shapes of the three parts of the mirror? [2]
(e) Will one kg of iron and one kg of cotton weigh same in air?
Justify your answer. [2]

Question 3 [10]
(a) State any two requisites for a choice of a unit. [2]
(b) Ripples are generated by a vibrator on the surface of water
which covers a distance of 37.8 cm in 1.8 s. The distance
between consecutive crest and trough is 1.5 cm. Calculate the [2]
(i) wavelength of the generated waves.
(ii) wave velocity of the generated waves.
(c) State Archimedes’ principle. [2]
(d) When a force is applied on a body A, it produces an
acceleration of 30 m.s–2. When it is applied on a body B, it
produces an acceleration of 18 m.s–2. What is the ratio of the
mass of body A to that of B? [2]
(e) Define the term focus and focal length of a concave mirror. [2]

Question 4 [10]
(a) In front of a plane mirror, an object is placed at a distance of
0.5 m. If the mirror is moved away from the object through a
distance of 0.2 m, by how much distance will the image move? [2]

2
(b) When two pins are hung by their heads from the same pole of a
magnet, their pointed ends move apart. Explain. [2]
(c) Draw a graph to show the variation in density of water when it
is heated from 0°C to 10°C. [2]
(d) State one property and one application of ultrasound. [2]
(e) A square plate of side 10 m is placed horizontally 1.5 m below
the surface of water. Calculate the thrust on the plate due to
water column.
(g = 10 m.s–2 and Density of water = 1000 kg m–3) [2]

SECTION II (40 Marks)


Attempt any 4 out of 6 questions from this Section

Question 5 [10]
(a) [3]
(i) Which mirror can be used in vehicles as a rear – view mirror?
Give reason.
(ii) Give another use of the mirror mentioned in (i).

(b) Give three points of difference between distance and


displacement. [3]
(c)
(i) How are the following derived units related to the fundamental
units:
1) newton,
2) pascal,
[2]
(ii) The bob of a simple pendulum is hollow with a pinhole at its
lower end. It is filled with water and allowed to oscillate. How
does its time period vary with time? [2]

Question 6 [10]
(a) Find the resistance of the wire if 40 J of work is done in moving
4 C of charge through it in 10 s. [3]
(b) State any three laws of liquid pressure. [3]

3
(c)
(i) Define the term acceleration due to gravity (g). Give its S.I unit. [2]
(ii) How does the value of ‘g’ change as we go: [2]
(1) Above the surface of the earth?
(2) Below the surface of the earth?

Question 7 [10]
(a) With the help of suitable ray diagram, show the formation of
images by two plane mirrors kept perpendicular to each other. [3]
(b) What is a natural magnet? State any two limitations of natural
magnet. [3]
(c) A force acts for 2 seconds upon a body initially at rest. As a
result, the body acquires a velocity of 8 m.s–1. If the mass of the
body is 55 kg, determine: [4]
(i) Initial momentum.
(ii) Final momentum.
(iii) Magnitude of force applied.
(iv) Acceleration.

Question 8 [10]
(a) Draw the pattern of magnetic field lines near a bar magnet
placed with its north pole pointing towards geographical north
of the earth. Indicate the position of neutral points by marking
‘X’. [3]
(b) A box weighing 5N has it’s base area of 5 cm 2. What pressure
does it exert? Another identical box is placed on top of the first
box. What is the thrust and pressure now? [3]
(c) The diagram below shows the displacement – distance graph
of a wave.
(i) State the values of amplitude and wavelength. [2]
(ii) If the velocity of the wave is 160 m.s-1, calculate the frequency. [2]

4
Question 9 [10]
(a)
(i) What do you mean by greenhouse effect? [1]
(ii) Name any 2 greenhouse gases. [2]
(b) Give three points of difference between primary cell and
secondary cell. [3]
(c)
(i) State Newton’s first law of motion. [2]
(ii) Why is it difficult to set a loaded trolley in motion than an
unloaded trolley? [2]

Question 10 [10]
(a)
(i) A body is moving with uniform velocity of 10 m/s for 10 s. What
will be its acceleration after 2 s? [1]
(ii) A body weighs 82.2 gf in air, 75.2 gf in water and 73.8 gf in a
liquid. Find the relative density of the liquid. [2]
(b)
(i) What do you mean by anomalous expansion of water? [1]
(ii) Explain how this phenomenon helps aquatic animals to survive
in frozen ponds. [2]
(c) [4]
(i) State the S. I. unit of resistance.
(ii) Diagrammatically illustrate how you would connect a plug key,
a cell, a voltmeter ‘V’, an ammeter ‘A’, an unknown resistance
‘R’ and conducting wires to find the unknown resistance.

*****

5
VIBGYOR HIGH
Sample Paper
TECHNICAL DRAWING APPLICATIONS
Grade: IX Max. Marks: 100
Date: Time Allowed: 3 hours

INSTRUCTIONS:-

 Answers to this paper must be written on the paper provided separately.


 You will not be allowed to write during the first 15 minutes.
 This time is to be spent in reading the question paper.
 The time given at the head of this paper is the time allowed for writing
the answers.
 The intended marks for the questions or parts of questions are given
alongside the questions.
 Separate sheets for section A and section B. Draw on both sides of the sheet.
 Neat and clean work is expected.

SECTION A
(Attempt any three questions)
Q.1
Using the third angle method of projection, draw the F,V , T.V & R.H.S.V & insert any six
dimensions. [16]

1
Q.2
a) Draw a hexagon of side 40mm.Inscribe three circles such that each circle will
touch each other & touches one side of the hexagon. [8]

b) Draw a triangle whose area is equal to sum of Isosceles triangle (Base = 55mm &
altitude =75mm) & equilateral triangle (of side = 45mm). [8]

Q.3
Copy the given template drawing. Retain all constructions neatly. [16]

Q.4
Draw an oblique view of the given object. Retain all constructions neatly. [16]

2
Q.5
Draw an isometric view of the given object. Retain all constructions neatly. [16]

SECTION B
(Attempt any two questions)

Q.6
Using first angle method of projection draw SEC. F.V along A-B, SEC T.V along
P-Q T.V & L.H.S.V & insert any six dimensions. [26]

3
Q.7
Draw an isometric view of the given object. Retain all constructions neatly. [26]

Q.8
a) Draw a circle of radius R=35mm, r = 20mm & CTC distance =109mm. Draw DCT
& TCT to given circles. [10]

b) Draw an ellipse by using ‘Rectangle method‘.


GIVEN: Major axis =150mm & Minor axis = 90mm [10]

c) Draw a pentagon of side 45mm.Convert into a triangle having same area. [6]

*****

4
Second Term Examination
AY 2020-2021
SUBJECT
SECOND LANGUAGE
Grade: IX Marks :80
Date :18/02/2021 Time Allowed:3 hour

INSTRUCTIONS:
• Answers to this paper must be written on the paper provided separately.
• You will not be allowed to write during the first 15 minutes.
• This time is to be spent in reading the question paper.
• The time given at the head of this paper is the time allowed for writing
the answers.
• The intended marks for the questions or parts of questions are given
alongside the questions.
• This question paper consists of 9 pages.
________________________________________________________________________

Section I (40 marks)


Attempt all questions from this section

Question.1

[15]
Write a short composition in Kannada of approximately 250
words on any
one of the following topics:
ಕೆಳಗೆ ಕೆೊಟ್ಟಿರುವ ವಿಷಯಗಳಲ್ಲಿ ನಿಮಗೆ ಬೆೇಕಾದ ಒಂದು ವಿಷಯವನ್ುು

ಆರಿಸಿಕೆೊಂಡು ಸುಮಾರು 250 ಪದಗಳ ಸಂಕ್ಷಿಪತ ಪರಬಂಧ ಬರೆಯಿರಿ:

(i) ‘‘ತಂದೆ ತಾಯಿ ಇಬ್ಬರು ಕಣ್ಣಿಗೆ ಕಾಣುವ ದೆೇವರು’’ ಈ ವಾಕಯದ ಸತಯತೆಯನ್ುು ಅರಿತು

ತಂದೆ ತಾಯಿಯನ್ುು ದೆೇವರಿಗೆ ಹೆ ೇಲಿಸಿರುವುದರ ಬ್ಗೆೆ ನಿಮ್ಮ ಮಾತುಗಳಲಿಿ ಒಂದು

ಪ್ರಬ್ಂಧ ರಚಿಸಿ.

1
(ii) “ಗಿಡವಾಗಿ ಬ್ಗೆದುು ಮ್ರವಾಗಿ ಬ್ಗಿೆೇತೆೇ?” ಈ ಗಾದೆಯನ್ುು ವಿಸತರಿಸಿ. ಈ ಗಾದೆ

ಮಾತನ್ುು ಆಧರಿಸಿ ಒಂದು ಕಥೆಯನ್ುು ಬ್ರೆಯಿರಿ.

(iii) “ಮ್ಳೆಯೇ ಜೇವನಾಧರ,ಮ್ಳೆ ಇಲ್ಿದೆ ಬೆಳೆ ಇಲ್ಿ.” ಮ್ಳೆಗಾಲ್ದ ಸಂದರ್ಭದಲಿಿ ಜನ್ರ

ಜೇವನ್ ಹೆೇಗಿರುತತದೆ. ಮ್ಳೆಯ ಅನ್ುಕ ಲ್ ಮ್ತುತ ಅನ್ನ್ುಕ ಲ್ಗಳ ಬ್ಗೆೆ ಕುರಿತಂತೆ

ಪ್ರಬ್ಂಧ ರಚಿಸಿ.

(iv) “ತಂತರಜ್ಞಾನ್ದ ಬೆಳವಣ್ಣಗೆ ಶಿಕ್ಷಣ ಕ್ೆೇತರದಲಿಿ ತಂದ ಬ್ದಲಾವಣೆ’’ ಇದು ಇಂದಿನ್ ಶಿಕ್ಷಣ

ಪ್ದಧತಿಗೆ ಹೆೇಗೆ ಪ್ೂರಕವಾಗಿದೆ ಅಥವಾ ಮಾರಕವಾಗಿದೆ ಎಂಬ್ುದನ್ುು ನಿಮ್ಮ

ಮಾತುಗಳಲಿಿ ಒಂದು ಪ್ರಬ್ಂಧ ರಚಿಸಿ.

(v) ಕೆಳಗೆ ಕೆ ಟ್ಟಿರುವ ಚಿತರವನ್ುು ಜಾಗರ ಕರಾಗಿ ಅಧಯಯನ್ ಮಾಡಿ, ಈ ಚಿತರವನ್ುು

ಆಧರಿಸಿ ಒಂದು ಕಥೆ ಅಥವಾ ಪ್ರಬ್ಂಧ ಬ್ರೆಯಿರಿ. ಚಿತರಕ ೂ ನಿಮ್ಮ ವಣಭನೆಗ

ಸಪಷ್ಿವಾದ ಸಂಬ್ಂಧವಿರಬೆೇಕು.

Question.2
Write a letter in Kannada approximately 120 words on any [7]
one of the topics
given below:
ಕೆಳಗೆ ಕೆೊಟ್ಟಿರುವ ವಿಷಯಗಳಲ್ಲಿ ನಿಮಗೆ ಬೆೇಕಾದ ಒಂದು ವಿಷಯವನ್ುು

ಆರಿಸಿಕೆೊಂಡು ಸುಮಾರು 120 ಪದಗಳಲ್ಲಿ ಒಂದು ಪತ್ರವನ್ುು ಬರೆಯಿರಿ:

(i) ‘’ಕಲೆಗಳ ಬೇಡು ನ್ಮ್ಮ ಕರುನಾಡು. ಈ ನಾಡು ವಾಸುತಶಿಲ್ಪಗಳ ತವರು.’’ ನಿೇವು ಭೆೇಟ್ಟ

ನಿೇಡಿದ ಐತಿಹಾಸಿಕ ದೆೇವಾಲ್ಯದ ಅನ್ುರ್ವಗಳನ್ುು ವಣ್ಣಭಸಿ ನಿಮ್ಮ ಗೆಳೆಯ /

ಗೆಳತಿಗೆ ಂದು ಪ್ತರ ಬ್ರೆಯಿರಿ.

2
ಅಥವಾ

(ii) ಸಾರಿಗೆ ವಯವಸೆೆ ನಿತಯಜೇವನ್ದಲಿಿ ಜನ್ರ ಪ್ರಯಾಣಕೊ ಅಗತಯವಾಗಿ ಬೆೇಕಾಗಿರುವ

ವಯವಸೆೆ. ಆದರೆ ನಿಮ್ಮ ಬ್ಡಾವಣೆಯಲಿಿ ಸರಿಯಾದ ಸಮ್ಯಕೊ ಸಾರಿಗೆ ವಯವಸೆೆ ಇಲ್ಿದ

ಕಾರಣ ಜನ್ಗಳಿಗೆ ಅನ್ನ್ುಕ ಲ್ತೆ ಉಂಟಾಗಿದೆ. ಈ ಸಮ್ಸೆಯಯನ್ುು ಕುರಿತಂತೆ

ಸಂಬ್ಂಧಪ್ಟ್ಿ ಸಾರಿಗೆ ಅಧಿಕಾರಿಗಳಿಗೆ ಒಂದು ಪ್ತರ ಬ್ರೆಯಿರಿ.

Question.3

Read the following passage carefully and answer in [10]


Kannada the questions
that follow:
ಕೆಳಗೆ ಕೆೊಟ್ಟಿರುವ ಗದಯ ಭಾಗವನ್ುು ಓದಿ, ಆನ್ಂತ್ರ ಕೆೇಳಲಾದ ಪರಶ್ೆುಗಳಿಗೆ ಉತ್ತರ

ಬರೆಯಿರಿ:

ಭಾರತದಲಿಿ ಪ್ುರಾತನ್ ಕಾಲ್ದಿಂದಲ್ ದೆೇಶಿಯ ಬಾಯಂಕುಗಳ ಮ್ ಲ್ಕ

ಅನೌಪ್ಚಾರಿಕ ಸಾಲ್ ನಿೇಡುವ ವಯವಸೆೆ ಇದೆ. ಅನೌಪ್ಚಾರಿಕ ಸಾಲ್ ವಲ್ಯವು

ದೆೇಶಿಯ ಬಾಯಂಕರರುಗಳನ್ುು ಒಳಗೆ ಂಡಿದೆ. ದೆೇಶಿಯ ಬಾಯಂಕರರನ್ುು ಶ್ಾರಫರು,

ಮ್ಹಾಜನ್ರು, ಶ್ೆೇಟ್ರು, ಸಾಹುಕಾರರು, ಚೆಟ್ಟಪಯಾರರು ಮ್ುಂತಾದ ಹೆಸರುಗಳಿಂದ

ಕರೆಯುತಾತರೆ. ಅವರು ಮ್ ಲ್ತಃ ಹಣದ ಲೆೇವಾದೆೇವಿದಾರರರಾಗಿದಾುರೆ. ಅವರ

ಜೆ ತೆಗೆ ವಾಯಪಾರಗಾರರು, ಉದೆ ಯೇಗದಾತರು ಸಂಬ್ಂಧಿಕರು ಮ್ತುತ ಸೆುೇಹಿತರು

ಸಹ ಸಾಲ್ ನಿೇಡುತಿತದುು, ಅವರ ಈವಲ್ಯದ ಭಾಗವಾಗಿದಾುರೆ.

೧೯ ನೆೇ ಶತಮಾನ್ದ ಮ್ಧಯಭಾಗದವರೆಗೆ ಭಾರತದಲಿಿ ದೆೇಶಿಯ ಬಾಯಂಕುಗಳು

ಹಣಕಾಸು ವಯವಸೆೆಯ ಕೆೇಂದರ ಭಾಗವಾಗಿದುವು. ಯುರೆ ೇಪಿನ್ ಬಾಯಂಕರರ

ಆಗಮ್ನ್ದಿಂದ ದೆೇಶಿಯ ಬಾಯಂಕರರ ಏಕಸಾಾಮ್ಯವು ತೆ ಂದರೆಗಿೇಡಾಯಿತು.

ಏಕೆಂದರೆ ಯುರೆ ೇಪಿನ್ ಬಾಯಂಕರರು ಭಾರತದಲಿಿ ಸಕಾಭರದ ರಕ್ಷಣೆ ಪ್ಡೆದಿದುರು.

ಆದಾಗ ಯ ಹಣದ ಲೆೇವಾದೆೇವಿದಾರರು ಸಾಲ್ ನಿೇಡಿಕೆಯಲಿಿ ಪ್ರಮ್ುಖ ಪಾತರ

ವಹಿಸುತಿತದುರು. ಸಾಾತಂತಾರಾನ್ಂತರ ಭಾರತ ಸಕಾಭರವು ಸಾಲ್ ನಿೇಡುವ

3
ವಯವಸೆೆಯನ್ುು ಹಣದ ಲೆೇವಾದೆೇವಿದಾರರ ಹಿಡಿತದಿಂದ ಮ್ುಕತಗೆ ಳಿಸಿ ಹಲ್ವಾರು

ಪ್ರಯತುಗಳನ್ುು ಮಾಡಿದೆ. ಸಕಾಭರವು ೧೯೬೯ ರಲಿಿ ೧೪ ಮ್ತುತ ೧೯೮೦ ರಲಿಿ ೬

ವಾಣ್ಣಜಯ ಬಾಯಂಕುಗಳನ್ುು ರಾಷ್ಟ್ರೇಕರಣ ಮಾಡಿತು. ಗಾರಮೇಣ ಭಾಗದ ಜನ್ರ ಸಾಲ್ದ

ಅಗತಯಗಳನ್ುು ಪ್ೂರೆೈಸುವುದಕಾೂಗಿ ಪ್ರತೆಯೇಕವಾಗಿ ಪಾರದೆೇಶಿಕ ಗಾರಮೇಣ

ಬಾಯಂಕುಗಳನ್ುು ಸಾೆಪಿಸಿತು. ಆದರೆ ದುರದೃಷ್ಿವಶ್ಾತ್ ಗಾರಮೇಣ

ಲೆೇವಾದೆೇವಿದಾರರು ಈಗಲ್ ಗಾರಮೇಣ ಸಾಲ್ ನಿೇಡಿಕೆಯಲಿಿ ಗಣನಿೇಯ ಪಾಲ್ನ್ುು

ಪ್ೂರೆೈಸುತಿತದಾುರೆ.

ಹಣದ ಲೆೇವಾದೆೇವಿದಾರರು ಗಾರಮೇಣ ಮ್ತುತ ನ್ಗರ ಪ್ರದೆೇಶಗಳಲಿಿ ಸಣಿ ಮ್ತುತ ಬ್ಡ

ಸಾಲ್ಗಾರರಿಗೆ ಸುಲ್ರ್ವಾಗಿ ಲ್ರ್ಯವಾಗುತಾತರೆ. ಇದೆೇ ಅವರು ಯಶಸಿಾಯಾಗಿ ತಮ್ಮ

ವಯವಹಾರ ಮ್ುಂದುವರಿಸಲ್ು ಕಾರಣವಾಗಿದೆ. ಗಾರಮೇಣ ಪ್ರದೆೇಶದಲಿಿ ಇವರ

ವಯವಹಾರ ಹೆಚಾಾಗಿಯೇ ಇದೆ. ಅವರು ಅಜ೯ಯ ಮೇಲೆ ಸಹಿ ಪ್ಡೆದು, ತಕ್ಷಣ ಸಾಲ್

ನಿೇಡುತಾತರೆ. ಆದರೆ ಲೆೇವಾದೆೇವಿದಾರರು ಅತಿಹೆಚಿಾನ್ ಬ್ಡಿಿದರವನ್ುು ವಿಧಿಸುತಾತರೆ.

ಇದೆೇ ಇವರ ಬ್ಹುದೆ ಡಿ ನ್ ಯನ್ಯತೆ. ಕೆಲ್ವು ವೆೇಳೆ ಇವರು ದಿನ್ದ ಮ್ತುತ ವಾರದ

ಆಧಾರದ ಮೇಲೆ ಬ್ಡಿಿ ವಿಧಿಸುತಾತರೆ. ಬ್ಡಿಿದರವೂ ಶ್ೆೇ.೩೪ ರಿಂದ ಶ್ೆೇ ೧೦೦ ವರೆಗೆ

ಅಥವಾ ಅದಕ್ೂಂತಲ್ ಹೆಚಿಾರುತತದೆ. ಇದು ಸಾಲ್ಗಾರನ್ ಹಣಕಾಸಿನ್ ತುತಭನ್ುು

ಅವಲ್ಂಭಿಸಿರುತತದೆ. ಬ್ಹಳಷ್ುಿ ಸಾಲ್ಗಾರರು ಸಾಲ್ದ ಬ್ಲೆಗೆ ಸಿಲ್ುಕ್ದಾುರೆ. ಅವರಿಂದ

ಏರುತಿತರುವ ಬ್ಡಿಿಯನ್ ು ಸಹ ಮ್ರುಪಾವತಿಸಲಾಗುತಿತಲ್ಿ.

ಪರಶ್ೆುಗಳು:

(i) ದೆೇಶಿಯ ಬಾಯಂಕರರನ್ುು ಯಾವ ಯಾವ ಹೆಸರುಗಳಿಂದ ಕರೆಯುತಾತರೆ? [2]

(ii) ದೆೇಶಿಯ ಬಾಯಂಕರರ‌ಏಕಸಾಾಮ್ಯಕೊ ತೆ ಂದರೆಯಾಗಲ್ು ಕಾರಣವೆೇನ್ು? ಭಾರತದಲಿಿ [2]

ಯಾವ ಬಾಯಂಕುಗಳು ಹಣಕಾಸು ವಯವಸೆೆಯ ಕೆೇಂದರ ಭಾಗವಾಗಿದುವು?

(iii) ಸಕಾಭರವು ಎಷ್ುಿ ಬಾಯಂಕುಗಳನ್ುು ಯಾವಾಗ ರಾಷ್ಟ್ರೇಕರಣ ಮಾಡಿತು? [2]

(iv) ಹಣದ ಲೆೇವಾದೆೇವಿದಾರರು ಯಾರಿಗೆ ಸುಲ್ರ್ವಾಗಿ‌ಲ್ರ್ಯವಾಗುತಾತರೆ? ಇವರು ಹೆೇಗೆ [2]

4
ಸಾಲ್ ನಿೇಡುತಾತರೆ?

(v) ಲೆೇವಾದೆೇವಿದಾರರ ಬ್ಹುದೆ ಡಿ ನ್ ಯನ್ಯತೆ ಯಾವುದು? ಇವರು ಯಾವ ರಿೇತಿಯ [2]

ಬ್ಡಿಿದರ ವಿಧಿಸುತಾತರೆ?

Question.4

Answer the following questions according to the [8]


instructions given:
ಕೆಳಗಿನ್ ಪರಶ್ೆುಗಳಿಗೆ ಸೊಚನೆಗಳಿಗೆ ಅನ್ುಸಾರವಾಗಿ ಉತ್ತರ ಬರೆಯಿರಿ:

(i) ಈ ವಾಕಯವನ್ುು ಅನ್ಯಲಿಂಗಕೊ ಬ್ದಲಿಸಿ ಬ್ರೆಯಿರಿ: [1]

ಅಗಸನ್ು ಬ್ಟೆಿಯನ್ುು ಒಗೆದನ್ು.

(ii) ಈ ಕೆಳಗಿನ್ ಯಾವುದಾದರ ಎರಡು ಪ್ದಗಳಿಗೆ ಸಮಾನಾಥಭಕ ಪ್ದಗಳನ್ುು [1]

ಬ್ರೆಯಿರಿ:

ಮ್ಂಕ, ಧಾರುಣ್ಣ, ದೆ ರೆ, ಸತಿ

(iii) ಈ ಕೆಳಗಿನ್ ವಾಕಯವನ್ುು ರ್ವಿಷ್ಯತಾೂಲ್ದಲಿಿ ಬ್ರೆಯಿರಿ: [1]

ಶ್ಾಲಿನಿ ಬ್ಹಳ ಸೆ ಗಸಾಗಿ ನ್ೃತಯವನ್ುು ಮಾಡಿದಳು.

(iv) ಈ ಕೆಳಗಿನ್ ವಾಕಯವನ್ುು ಬ್ಹುವಚನ್ಕೊ ಬ್ದಲಿಸಿ ಬ್ರೆಯಿರಿ; [1]

ಕೆ ೇತಿ ಹಣಿನ್ುು ತಿಂದಿತು.

(v) ಈ ಕೆಳಗಿನ್ ಯಾವುದಾದರ ಎರಡು ಪ್ದಗಳಿಗೆ ವಿರುದಾಧಥಭಕ ಪ್ದಗಳನ್ುು [1]

ಬ್ರೆಯಿರಿ:

ನಿರುತಾಾಹ, ಪಾಪ್, ಇಹ, ಸಿೆರ

(vi) ಬಟ್ಿ ಸೆಳಗಳಲಿಿ ಸ ಕತವಾದ ವಿರ್ಕ್ತ ಪ್ರತಯಯಗಳನ್ುು ಬ್ರೆಯಿರಿ: [1]

ತಂದೆ-ತಾಯಿಯರಿ...... ಪ್ರತಿದಿನ್ ನ್ಮ್ಸಾೂರ........ ಮಾಡಬೆೇಕು.

(vii) ಈ ಗಾದೆ ಮಾತನ್ುು ಪ್ೂಣಭಗೆ ಳಿಸಿ: [1]

ಸತಯಕೊ ಸಾವಿಲ್ಿ,‌…………………….

(viii) ಈ ಕೆಳಗಿನ್ ಯಾವುದಾದರ ಎರಡು ಪ್ದಗಳಿಗೆ ನಿಮ್ಮ ಸಾಂತ ವಾಕಯಗಳನ್ುು ರಚಿಸಿ. [1]

5
ವಿಧಿ, ಸಂದೆೇಹ, ಅಸಮಾಧಾನ್, ಗೌರವ

SECTION – B (40 Marks)

Attempt four questions from Section You must answer at least


one question each from the two books you have Studied and
any two other questions.
ಸಾಹಿತ್ಯ ಸಂಗಮ – ಸಣ್ಣಕಥೆಗಳು

SAAHITYA SANGAMA - PROSE


Question.5
Read the extract given below and answer in Kannada the [10]
questions that follow:
ಕೆಳಗೆ ಕೆೊಟ್ಟಿರುವ ಗದಯಭಾಗವನ್ುು ಓದಿ, ಆನ್ಂತ್ರ ಕೆೇಳಲಾದ ಪರಶ್ೆುಗಳಿಗೆ ಕನ್ುಡದಲ್ಲಿ

ಉತ್ತರಿಸಿ:

ನಾನ್ು ಏನ್ು ಮಾಡಬೆೇಕೆಂದು ತೆೊೇರದೆ ಎರಡು ನಿಮಿಷ ಅಲೆಿೇ ಸುಮಮನೆ ನಿಂತ್ುಬಿಟ್ೆಿ

ಎರಡು ನಿಮಿಷಗಳಲ್ಲಿ ನ್ೊರಾರು ಜನ್ ನ್ನ್ು ಎದುರಿಗೆ ಹೆೊೇದರು ಅವರೆಲ್ಿರಿಗೊ ನ್ನ್ು

ಗರುಡಗಂಬದ ಮೇಲೆ ದೃಷ್ಟಿಯಿದಿಿತೆೇ ಹೆೊರತ್ು ಮತಾತವುದರ ಮೇಲ್ೊ ಇರಲ್ಲಲ್ಿ.

ಗರುಡಗಂಬದ ದಾಸಯಯ- ಗೆೊರೊರು ರಾಮಸಾಾಮಿ ಅಯಯಂಗಾರ್

Garudagambada Dasayya - Gorooru Ramaswamy Aiyangar


(i) ಲೆೇಖಕರ ವೆೇಷ್ರ್ ಷ್ಣ ಹೆೇಗಿತುತ? [2]

(ii) ಸೆುೇಹಿತನ್ ಕೆ ಠಡಿಯಲಿಿದವ


ು ರಲಿಿ ಒಬ್ಬ ಲೆೇಖಕರ ಗೆಳೆಯನಿಗೆ ಏನೆಂದು ಬೆೈದನ್ು? [2]

(iii) ಗೆ ರ ರು ರಾಮ್ಸಾಾಮ ಅಯಯಂಗಾರ್ ಅವರ ಸಂಕ್ಷಿಪ್ತ ಪ್ರಿಚಯ ಬ್ರೆಯಿರಿ. [3]

(iv) ಕೃಷ್ಿ ಬ್ಜಾರಿನ್ ಮ್ುಂದೆ ಜನ್ರು ಲೆೇಖಕರನ್ುು ನೆ ೇಡುತಿತದಾುಗ ಅವರ ಮ್ನ್ದಲಿಿ [3]

ಮ್ ಡಿದ ಯೇಚನೆಗಳೆೇನ್ು?

Question.6

Read the extract given below and answer in Kannada the [10]
questions that follow:
ಕೆಳಗೆ ಕೆೊಟ್ಟಿರುವ ಗದಯಭಾಗವನ್ುು ಓದಿ, ಆನ್ಂತ್ರ ಕೆೇಳಲಾದ ಪರಶ್ೆುಗಳಿಗೆ ಕನ್ುಡದಲ್ಲಿ

6
ಉತ್ತರಿಸಿ:

ಆಗಲೆೇ ಹೆೊತೆತೇರಿತ್ುತ. ಹುಡುಗ ಬೆೇಗ ಬೆೇಗ ಹೆಜ್ೆೆ ಇಡುತ್ತತದ.ಿ ಇಷುಿ ವೆೇಳೆಗಾಗಲೆೇ

ಬಿೇಡಿ ಫ್ಾಯಕಿರಿಯಲ್ಲಿ ಇರಬೆೇಕಿತ್ುತ. ಆದರೆ ದಾರಿಯಲ್ಲಿ ಗೆೊೇಲ್ಲ ಆಡುತ್ತತದಿ ಗೆಳೆಯರ

ಗುಂಪಿನ್ಲ್ಲಿ ಬೆರೆತ್ು ತಾನ್ು ಗೆೊೇಳಲ್ಲ ಆಡಿದ.

ತ್ುಚೇಪ್, ತ್ುದಾಂಡ್, ತ್ುಬತ್ - ರೆಡಿೇ – ಚದುರಂಗ

TUCHIP, TUDAAND, TUBAT-REDI- CHADURANGA

(i) ಕರಿೇಮ್ ನ್ನ್ುು ಅವರ ತಂದೆ ಶ್ಾಲೆ ಬಡಿಸಲ್ು ಕಾರಣವೆೇನ್ು? [2]

(ii) ಕರಿೇಮ್ ಬೇಡಿ ಕಾರ್ಾಭನೆಯಲಿಿ ಮಾಡುತಿತದು ಕೆಲ್ಸವೆೇನಾಗಿತುತ? ವಿವರಿಸಿ. [2]

(iii) ತಿಮಾಮರೆಡಿಿ ಸವಾಲ್ನ್ುು ಸಿಾೇಕರಿಸಿದ ಕರಿೇಮ್ ಗೆಲ್ುವು ಸಾಧಿಸಿದ ಬ್ಗೆಯನ್ುು [3]

ವಿವರಿಸಿ.

(iv) ಕರಿೇಮ್ ನಿಗೆ ಜನಾಧಭನ್ನ್ ಮೇಲೆ ಅಸ ಯ ಉಂಟಾಗಲ್ು ಕಾರಣವೆೇನ್ು? [3]

Question.7
Read the extract given below and answer in Kannada [10]
thequestions that follow:
ಕೆಳಗೆ ಕೆೊಟ್ಟಿರುವ ಗದಯಭಾಗವನ್ುು ಓದಿ, ಆನ್ಂತ್ರ ಕೆೇಳಲಾದ ಪರಶ್ೆುಗಳಿಗೆ ಕನ್ುಡದಲ್ಲಿ

ಉತ್ತರಿಸಿ:

ತ್ತೇಥಥಯಾತೆರಗೆ ಹೆೊರಟ ದಯಾಮಯನಾದ ಶಿವಭೊತ್ತ ಎಂಬ ಬಾರಹಮಣ್ನ್ು

ಬಾಯಾರಿ ಅದೆೇ ಬಾವಿಗೆ ನಿೇರು ಕುಡಿಯಲ್ು ಬರುತಾತನೆ. ಆ ಬಾವಿಯಿಂದ ನಿೇರು

ಪಡೆಯಲ್ು ಒಂದು ಬಳಿಿಗೆ ತ್ನ್ು ಸಣ್ಣ ಕಳಶ ಗಿಂಡಿಯನ್ುು ಕಟ್ಟಿ ಬಾವಿಯಲ್ಲಿ ಬಿಡುತಾತನೆ.

ಶಿವಭೊತ್ತಯ ಕಥೆ – ಪಂಚತ್ಂತ್ರ

Shivabhootiya Kathe – Pancha Tantra


(i) ಹುಲಿ ಮ್ತುತ ಕಪಿ ಶಿವರ್ ತಿಯನ್ುು ಹೆೇಗೆ ಸತೂರಿಸಿದವು? [2]

(ii) ಪ್ಂಚತಂತರದಲಿಿ ಅಡಕವಾಗಿರುವ ಅಂಶಗಳು ಯಾವುವು? [2]

(iii) ಹುಲಿಯು ಶಿವರ್ ತಿಗೆ ಏನೆಂದು ಹೆೇಳಿತು? [3]

(iv) ಶಿವರ್ ತಿಗೆ ಸಪ್ಭವು ಹೆೇಗೆ ನೆರವಾಯಿತು? [3]

7
ಸಾಹಿತ್ಯ ಸಂಗಮ -ಕವನ್ಗಳು

SAAHITYA SANGAMA - POETRY

Question.8

Read the extract given below and answer in Kannada the [10]
questions that follow:
ಕೆಳಗೆ ಕೆೊಟ್ಟಿರುವ ಗದಯಭಾಗವನ್ುು ಓದಿ, ಆನ್ಂತ್ರ ಕೆೇಳಲಾದ ಪರಶ್ೆುಗಳಿಗೆ ಕನ್ುಡದಲ್ಲಿ

ಉತ್ತರಿಸಿ:

ಅನ್ಯರಲ್ಲ ಜನಿಸಿದ೯ ಅಂಗನೆಯ ಕರೆತ್ಂದು

ತ್ನ್ು ಮನೆಗವಳ ಯಜಮಾನಿಯೆನಿಸಿ

ಭಿನ್ುವಿಲ್ಿದ ಅಧಥ ದೆೇಹ ವೆನಿಸುವ ಸತ್ತಯು

ಕಣ್ಣಣನ್ಲ್ಲಿ ನೆೊೇಡಲ್ಮಮಳು ಕಾಲ್ನೆೊಯಾಾಗ

ಆರು ಹಿತ್ವರು ನಿನ್ಗೆ - ಪುರಂದರ ದಾಸರು

AARU HITAVARU NINAGE

(i) ಪ್ುರಂದರದಾಸರ ಪ್ೂವಭದ ಹೆಸರೆೇನ್ು? ಗುರುಗಳ ಹೆಸರಿನೆ ಂದಿಗೆ ಸಂಕ್ಷಿಪ್ತವಾಗಿ [2]

ಪ್ರಿಚಯಿಸಿ.?

(ii) ಮ್ನ್ುಜರಿಗೆ ದಾಸರ ಸಂದೆೇಶವೆೇನ್ು? [2]

(iii) ಯಾವ ಯಾವ ರಿೇತಿಯಲಿಿ ಸಂಪ್ತತನ್ುು ಗಳಿಸಬ್ಹುದು ಎಂದು ದಾಸರು ಹೆೇಳಿದಾುರೆ? [3]

(iv) ನಾರಿಯ ಬ್ಗೆೆ ವಾಯಮೇಹ ಕುರಿತು ದಾಸರು ಏನ್ು ಹೆೇಳಿದಾುರೆ? [3]

Question.9
Read the extract given below and answer in Kannada the [10]
questions that follow:
ಕೆಳಗೆ ಕೆೊಟ್ಟಿರುವ ಗದಯಭಾಗವನ್ುು ಓದಿ, ಆನ್ಂತ್ರ ಕೆೇಳಲಾದ ಪರಶ್ೆುಗಳಿಗೆ ಕನ್ುಡದಲ್ಲಿ

ಉತ್ತರಿಸಿ:

ಯಾರೊ ಅರಿಯದ ನೆೇಗಿಲ್ ಯೇಗಿಯೆ

8
ಲೆೊೇಕಕೆ ಅನ್ುವನಿೇಯುವನ್ು

ಹೆಸರನ್ು ಬಯಸದೆ ಅತ್ತಸುಖಕೆಳಸದೆ

ನೆೇಗಿಲ್ ಯೇಗಿ – ಕುವೆಂಪು

NEGILA YOGI - KUVEMPU

(i) ನೆೇಗಿಲ್ಯೇಗಿ ಯಾರು? ಅವನ್ು ಹೆೇಗೆ ದುಡಿಯುವನ್ು? [2]

(ii) ಲೆ ೇಕದಲಿಿ ಏನೆೇ ನ್ಡೆಯುತಿತದುರ ರೆೈತ ಏನ್ು ಮಾಡುವನ್ು? [2]

(iii) ರೆೈತರನ್ುು ಕವಿ ‘ಸೃಷ್ಟ್ಿಯ ನಿಯಮ್ದೆ ಳು ಭೆ ೇಗಿ’ ಎಂದು ಏಕೆ ಕರೆದಿದಾುರೆ? [3]

(iv) ನೆೇಗಿಲ್ನಾಶರಯದಿ ನಾಗರಿಕತೆ ಹೆೇಗೆ ಬೆಳೆಯಿತೆಂದು ಕವಿ ಹೆೇಳುತಾತರೆ? [3]

Question.10
Read the extract given below and answer in Kannada the [10]
questions that follow:
ಕೆಳಗೆ ಕೆೊಟ್ಟಿರುವ ಗದಯಭಾಗವನ್ುು ಓದಿ, ಆನ್ಂತ್ರ ಕೆೇಳಲಾದ ಪರಶ್ೆುಗಳಿಗೆ ಕನ್ುಡದಲ್ಲಿ

ಉತ್ತರಿಸಿ:

ನ್ಡೆದು ಯಾರ ಕೆೊರಳಿನ್ಲ್ಲಿ

ತ್ಡವರಿಸುವುದೆೊ ಅವರ ಪಟಿ

ಕೆೊಡೆಯರನ್ುು ಮಾಳೆೆವೆಂದು ಬಿಟಿರಲ್ಲಿಯೆ

ತ್ತರುಕನ್ ಕನ್ಸು - ಮುಪಿೆನ್ ಷಡಕ್ಷರಿ

TIRUKANA KANASU – MUPPINA SHADAKSHARI


(i) ಆಸಾೆನ್ದಲಿಿ ತಿರುಕ ತನ್ು ಮ್ಂತಿರಗೆ ಏನೆಂದು ಹೆೇಳಿದನ್ು? [2]

(ii) ತಿರುಕ ಹೆೇಗೆ ವಾಸತವತೆಗೆ ಮ್ರಳಿದನ್ು? [2]

(iii) ತಿರುಕನ್ ಹೆೇಗೆ ರಾಜಯವಾಳಿದನ್ು? [3]

(iv) ಈ ಪ್ದಯದ ನಿೇತಿಯನ್ುು ನಿಮ್ಮ ಮಾತುಗಳಲಿಿ ಬ್ರೆಯಿರಿ. [3]

*****

9
VIBGYOR HIGH
Sample Paper
ART PAPER - 1
DRAWING AND PAINTING FROM STILL LIFE
Grade: IX Max. Marks: 100
Date: Time Allowed: 3 hour
___________________________________________________________________
INSTRUCTIONS:
 Candidate's Index Number must be written clearly on the top right hand
corner of the front surface of the paper.
 Mechanical devices should not be used.
_____________________________________________________________________
Candidates may attempt either Theme A or Theme B
Theme A. [100]
Objects Required:
Thermos Bottle, Hand Wash Dispenser, Sanitizer, One Mask And One Handkerchief
Arrangements:
Do the arrangement as per good composition. A suitable background must be
included.

Theme B. [100]
Objects Required:
One Transparent Jar With Cornflakes, One Teacup With Saucer, Open Parle-G
Biscuit Packet, One steel spoon
Arrangements:
Do the arrangement as per good composition. A suitable background must be
included.

****************

This paper consists of 1 printed page.


VIBGYOR HIGH
Sample Paper
ART PAPER - 3
ORIGINAL IMAGINATIVE COMPOSITION IN COLOUR

Grade: IX Max. Marks: 100


Date: Time Allowed: 3 hour
___________________________________________________________________
INSTRUCTIONS:

 Candidate’s Index Number must be written clearly on the top right hand
corner of the front surface of the paper.
 You must not base your composition on any picture you have seen.
 Bear in mind that the quality of the composition, feeling and originality are
the main objectives of this paper.
 Mechanical devices should not be used.
_____________________________________________________________________
Attempt any one of the following topics: [100]

1. Marine life.
2. Any scene depiction from any sport with cheering and motivating audience.
3. Corona virus Pandemic - Changing Lives
4. Lines of a poem which stimulate imagination:
"Twinkle Twinkle little star.
How I wander what you are.
Up above the world so high,
Like a diamond in the sky".
Colour the composition with suitable colour scheme and medium.

****************

This paper consists of 1 printed page.


VIBGYOR HIGH
Sample Paper

ECONOMICS

Grade: IX Max. Marks: 80

Date: Time Allowed: 2 hours

INSTRUCTIONS:

 Answers to this paper must be written on the paper provided separately.


 You will not be allowed to write during the first 15 minutes.
 This time is to be spent in reading the question paper.
 The time given at the head of this paper is the time allowed for writing the
answers.
 The intended marks for the questions or parts of questions are given
alongside the questions.
 Attempt all questions from Section A and any four questions from
Section B.

SECTION A (40 Marks)


Attempt all questions from this section.
Question. Answer the following questions. [10]
1
a. Explain Non-Economic activity with help of an example. [2]

b. How is Absolute Poverty different from Relative poverty? [2]

c. Define a Laissez -Faire economy. [2]

d. Explain any two economic agents of an economy? [2]

e. How does improved health contribute to economic growth? [2]

1
Question. Answer the following questions. [10]
2
a. What are the contributions of agriculture to the economy? [2]

b. Give any two examples of economic activity. [2]

c. Which factors of production are passive in nature and why? [2]

d. Mention any two features of medium scale industries. [2]

e. What do you mean by economic growth? [2]

Question. Answer the following questions. [10]


3
a. What is seasonal unemployment? [2]

b. How will you classify an economy on basis of ownership of means of [2]


production?
c. Explain how HYV seeds can improve the agricultural production? [2]

d. Give the meanings of following terms: [2]


i) Savings
ii) Wants

e. State Growth definition of economics. [2]

Question. Answer the following questions. [10]


4
a. How does vulnerability indicate poverty? [2]

b. Explain the term food security. [2]

c. How does bank play an important role in industrial and commercial life [2]
of a country?

d. What is the meaning of ‘Mixed Economy’? [2]

e. Explain any two causes of problems of an economy. [2]

2
SECTION B
(Attempt any four questions from this section)

Question. a. i) Define industries. Give an example. [2]


5 ii) Discuss the impact of industrialisation on ecosystem. [3]

b. Explain the following concepts: [5]


i) Utility ii) Services iii) Production
iv) Market v) Economy

Question. a. Distinguish between a developed economy and developing economy. [5]


6
b. Enumerate the causes of Unemployment in India. [5]

Question. a. (i) Explain the term Consumption. [1]


7 (ii) Write a short note on value and price. [4]

b. Why is there a need of cottage and small-scale industries in Indian [5]


economy?

Question. a. Distinguish between Micro- Economics and Macro Economics. [5]


8
b. Explain the term ‘Poverty line’. Discuss the factors responsible for [5]
poverty in India.

Question. a. ‘The government has adopted many measures to improve agricultural [5]
9 production in India’. Explain.

b. (i) What are the various range of services available in tertiary sector? [1]
(ii) ‘Agriculture and Industries are interdependent on each other’. [4]
Explain.

Question. a. Enumerate the positive impacts of Green Revolution. [5]


10
b. Explain the role of communication in the economic development of a [5]
country.
*****

3
VIBGYOR HIGH
Sample Paper
BIOLOGY
SCIENCE Paper- 3
Grade: IX Max. Marks: 80
Date: Time Allowed: 2hours

IN INSTRUCTIONS: -
Answers to this Paper must be written on the paper provided separately.
You will not be allowed to write during the first 15 minutes.
This time is to be spent in reading the Question Paper.
The time given at the head of this Paper is the time allowed for writing the answers.
_________________________________________________________________________
Attempt all questions from Section I and any four questions from Section II.
The intended marks for questions or parts of questions are given in brackets [ ].
This Paper consists of 9 Printed pages.

SECTION I (40 Marks)


Attempt all questions from this section
Question1
a) Name the following: [5]
(i) Finger like folds in the inner membrane of mitochondria.
(ii) Protective coverings of the ovule.
(iii) Respiratory openings found on the stems of woody plant.
(iv) Mode of nutrition in which organisms draw nourishment from decaying dead
organisms.
(v) An unstable compound formed between haemoglobin and oxygen to carry
oxygen to the body cells.

1
(b) State whether the following statements are true or false. If false rewrite the [5]
correct statement by changing the first or the last word only.
(i) Cells are always dead in parenchyma.
(ii) Inflorescence is the manner in which the ovules are attached to the walls of
the ovary.
(iii) Micropyle stores food.
(iv) Amino acids are building blocks of fat.
(v) Cartilages are supporting and connecting tissues.

(c) Match the items in Column ‘A’ with those which are most appropriate in [5]
Column ‘B’. Rewrite the matching pairs as shown in the example:
Example: Patella – Knee joint.
Column A Column B
(i) Repair (a) Skin
(ii) Castor (b) Waste management
(iii) TAB vaccine (c) Tuberculosis
(iv) Composting (d) Hypogeal germination
(v) Melanin (e) Cotton
(f) Typhoid
(g) Cell division
(h) Epigeal germination

(d) Choose the ODD one out from the following terms given and name the [5]
CATEGORY to which the others belong:
(i) Lymph, Plasma, Vaccine, Serum
(ii) Unisexuality, Self-sterility, Autogamy, Dichogamy
(iii) Glycogen, Starch, Cellulose, Albumen
(iv) Ilium, Glenoid, Pubis, Ischium
(v) Granular layer, Germinative layer, Subcutaneous layer, Cornified layer

2
(e) Study the given diagram and answer the questions that follow: [5]

(i) Name the physiological process being studied.


(ii) Give an overall balanced chemical equation to represent the process.
(iii) What change, if any, would you observe in the lime water in flask A and B?
In each case give a reason for your answer.
(iv) Mention one precaution that should be taken to ensure more accurate
results.

(f) Choose the correct alternative from the four options given below: [5]
(i) Nitrates are replenished in the soil by the bacteria through the process of
(a) Nitrification
(b) Denitrification
(c) Fermentation
(d) Deamination

(ii) Which one of the following is found both in the cells of a papaya plant and a
monkey?
(a) Cell wall
(b) Centriole
(c) Chloroplast
(d) Chromatin fibres

3
(iii) The filaments are united in several groups in:
(a) Cotton
(b) Pea
(c) Bombax
(d) China rose

(iv) ____________ is a characteristic feature of inner lining of the intestine.


(a) Bolus
(b) Pilus
(c) Microvilli
(d) Glottis

(v) Small processes present in the outer region of dermis is called:


(a) Cilia
(b) Papillae
(c) Corneum
(d) Stratum

(g) Given below are sets of five terms each. Rewrite the terms in a correct [5]
orderso as to be in a logical sequence:
(i) Calyx, Androecium, Gynoecium, Epicalyx, Corolla
(ii) Zygote, Embryo, Seed, Allogamy, Fusion of gametes
(iii) Cardiac Sphincter, Small intestine, Chyme, Stomach, Pylorus
(iv) Trachea, Larynx, Alveoli, Bronchi, Pharynx
(v) Class, Kingdom, Phylum, Order, Family
(h) State the main function of the following: [5]
(i) Nucellus
(ii) Cuboidal epithelium
(iii) Penicillin
(iv) Sodium
(v) Malpighian layer

4
SECTION II (40 marks)

Attempt any four questions from this section

Question2
(a) Observe the given diagram and answer the following questions: [5]

(i) Label the parts 1 to 4.


(ii) State role of part labelled 1.
(iii) What is the fate of the part labelled 3 and 4 after fertilisation?
(iv) Define part labelled 6.

(b) Differentiate between the following pairs on the basis of what is given in the [5]
brackets:
(i) Herkogamy and Heterostyly (Meaning of the terms)
(ii) Cytoplasm and Cell sap (Function)
(iii) Anaerobic respiration in plants and Anaerobic respiration in animals (End
products)
(iv) Hinge joint and Ball and Socket joint(Range of movement)
(v) Anabolic process and Catabolic process (Type of process)

5
Question3
(a) Give two examples for each of the following: [5]
(i) Fluid connective tissue
(ii) Ornithophilous flowers
(iii) Chemicals which absorb carbon dioxide in respiratory experiments
(iv) Disinfectants
(v) Gliding joint

(b) Answer the following briefly: [5]


(i) What is triple fusion?
(ii) Anthrax is used as bio weapon. Why?
(iii) List any two uses of water for germination.
(iv) List two useful role of Fungi in food industry.
(v) State two functions of sebum.

Question4
(a) Study the diagram and answer the questions: [5]

(i) Label the parts marked 1 – 4.


(ii) Identify the plant part as a whole.
(iii) What is the function of part 3?
(iv) What happens to part labelled 4 during the process?

6
(b) Draw a diagram and answer the questions based on the diagram drawn. [5]
(i) Draw a neat diagram of a thoracic vertebra and label any four parts.
(ii) Give the role of articular facets.
(iii) Name the first and the second vertebra of vertebral column.

Question 5
(a) Study the diagram and answer the questions: [5]

(i) What is the aim of the experiment?


(ii) Why is disinfectant added in Test-tube Y?
(iii) Why has coloured water risen in tubing 1?
(iv) What will happen if burning paper is introduced into test-tube X and Y.
(v) What is a control set up?

(b) Explain the following terms: [5]


(i) Pandemic
(ii) Cleistogamy
(iii) Anaemia
(iv) Joint
(v) Innate immunity

7
Question 6
(a) Give the biological term for the following: [5]
(i) Removal of anthers in young flowers in artificial crossing.
(ii) Male and female flower grow on different plants.
(iii) The region of the axis between the point of attachment of cotyledons and the
plumule.
(iv) Conversion of the absorbed digested food into body material.
(v) The volume of air that can be taken in and expelled out by maximum
inspiration and expiration.
(vi) Whorl of the flower which protects the young flower bud and when green
they also perform photosynthesis.
(vii) Transfer of pollen from one flower to the stigma of another flower of the
same plant.
(viii) The ripened ovule that contains embryo inside it which develops into a new
plant.
(ix) Soft connective tissue rich in blood capillaries and nerve fibres, found in the
central space of the tooth.
(x) Air that can be forcibly expelled out after normal expiration.

(b) Draw a neat and well labelled diagram and answer the questions based on [5]
the diagram drawn.
(i) Draw a neat and well- labelled diagram of internal structure of a Bean seed
and label any two parts in the diagram drawn.
(ii) State any one function of a micropyle.
(iii) Very low and very high temperatures are unsuitable for germination. Justify.
(iv) What type of germination is seen in bean seeds?

8
Question 7
(a) Study the following diagram and answer the questions. [5]

(i) Identify state of breathing represented in the given figure.


(ii) What role does internal intercostal muscles and external intercostal muscles
play during the process shown in the diagram given above.
(iii) During the process shown above, what changes would take place in thoracic
cavity with respect to its size and air pressure?
(iv) Where is respiratory centre located? How does it control the breathing
movements?
(v) Explain the term dead air space. How much quantity of air is considered as
dead air space?

(b) Give reasons for the following statements. [5]


(i) Meristematic cells have a prominent nucleus and dense cytoplasm but they
lack vacuole.
(ii) Tilling of soil is useful for crops.
(iii) Five kingdom system of classification is preferred over two kingdom system
of classification.
(iv) Cellulose is not digested, yet it is an important component of a balanced
diet.
(v) Herbivores have no canine teeth.
*****

9
VIBGYOR HIGH
Sample Paper
CHEMISTRY
Grade: IX Max. Marks: 80
Date: Time allowed: 2 hours
___________________________________________________________

INSTRUCTIONS:
 Answers to this paper must be written on the paper provided separately.
 You will not be allowed to write during the first 15 minutes.
 This time is to be spent in reading the question paper.
 The time given at the head of this paper is the time allowed for writing the
answers.
 Attempt all questions from Section I and any four questions from section II.
 The intended marks for the questions or parts of questions are given
alongside in brackets [ ].
________________________________________________________________

SECTION-I (40 Marks)


Attempt all questions from this Section

Question 1.

A Fill in the blanks with choices given in the brackets: [5]

(i) The reaction of iron with steam, liberating hydrogen gas is a_______________
[irreversible/ reversible] reaction.
(ii) The valency of copper in cupric chloride is_____________ [1+/ 2+].
(iii) 1 dm3 of a gas is equal to ___________ [1 lit / 100 ml / 100 cc]
(iv) __________ is an example of efflorescent salt [iron (lll) chloride / washing
soda].
(v) A saturated solution can be converted to an unsaturated solution by _______
[increasing/decreasing] the amount of solvent.

1
B. Choose the most appropriate option: [5]
(i) The salt whose solubility decreases with rise in temperature is:
a) Calcium sulphate
b) Sodium nitrate
c) Potassium nitrate
d) Ammonium chloride.

(ii) The maximum number of electrons that can be present in the M-shell of an
atom:
a) 2
b) 8
c) 18
d) 32

(iii) Number of neutrons present in an atom with electronic configuration 2, 7 and


mass no.19:
a) 8
b) 10
c) 12
d) 7

iv) The salt that gives black residue on heating:


a) Lead carbonate
b) Copper carbonate
c) Calcium carbonate
d) Magnesium carbonate

v) Which of the following is a good oxidizing agent?


a) Chlorine
b) Aluminium
c) Potassium
d) Magnesium

2
C Select from A to K given in the list that matches the description given [5]
below:
[A.Sodium B.Carbon C.Magnesium D.Aluminium E.Chlorine F.Fluorine
G.Helium H.Period Number I.Group Number J.Neon K. Beryllium]
i) A noble gas with duplet arrangement of electrons.
ii) An alkaline earth metal in period 3.
iii) A tetravalent element in period 2.
iv) It signifies the number of valence electrons of an element.
v) The most reactive halogen.

D Match the names of the compounds in Column ‘A’ with their correct [5]
formulas in Column ‘B’:
Column A Column B
1. Phosphoric acid a. H2PO4
2. Potassium Chlorite b. CaO
3. Epsom salt c. H3PO4
4. Quick lime d. KClO2
5. Gypsum e. MgSO4.7H2O
f. CaSO4.2H2O
g. KCl

E. Write balanced chemical equations for the following reactions : [5]


i) Action of steam on heated aluminium metal.
ii) Action of dilute hydrochloric acid on granulated zinc.
iii) Action of hot conc. sodium hydroxide solution on lead.
iv) Action of heat on calcium carbonate.
v) Oxidation of metallic copper.

F. State one relevant observation when: [5]


i) Water is added to anhydrous copper sulphate.
ii) Salt of lead nitrate is strongly heated.
iii) A solution of sodium chloride is added to lead nitrate solution.
iv) Sodium metal is dropped into a beaker containing cold water.
v) Ammonium dichromate is strongly heated.

3
G. Give appropriate reasons for the following: [5]
i) Common salt turns moist on exposure to air.
ii) Gases are highly compressible.
iii) Group I A elements are called ‘Alkali metals’.
iv) Isotopes of chlorine have different physical properties.
v) Noble gases are extremely unreactive.

H. Solve the following: [5]


(i) A gas occupies 800 cm3 under 760 mm pressure. Find under what pressure
the gas will occupy 380 cm3, the temperature remaining constant.
ii) What temperature would be necessary to double the volume of a gas initially at
S.T.P, if the pressure is decreased by 50%?

Section II (40 marks)


Attempt any four questions from this section

Question 2.
A. Answer the following questions: [3]
i) Name an element that violently reacts with water at room temperature.
ii) What do the following symbols denote?
a) 2H b) H+
iii) Why is potassium not used to prepare hydrogen in the laboratory?

B. An atom is represented as 27
13X [3]
i) Write the electronic configuration of ‘X’.
ii) Mention the number of neutrons present in it.
iii) State whether ‘X ’is a metal or a non- metal.

C. Classify the following reactions as : [4]


a) Direct combination b) Decomposition c) Displacement
d) Neutralisation e) Precipitation
(i) Pb(NO3)2 + 2NaCl -------------- PbCl2 + 2NaNO3
(ii) CaO + H2O -------------- Ca(OH)2

4
(iii) NH4OH + HCl -------------- NH4Cl + H2O
(iv) 2AgNO3 -------------- 2Ag + 2NO2 +O2

Question 3.
A. Draw the orbital diagram of the following compounds/ molecules and
[4]
mention the type of bond formed in each of them:
i) Sodium chloride
ii) Nitrogen molecule

B. Distinguish between the following: ( one point of difference on the basis [4]
of definition and an example)
i) Electrochemical reaction / Photochemical reaction
ii) Oxidation reaction / Reduction reaction

C. Write balanced chemical equations for the following word equations: [2]
i) Aluminium + Oxygen -------------- Aluminium oxide
ii) Iron + Chlorine ------------ Iron (llI) chloride

Question 4.
A. Give reason for the following with respect to the laboratory preparation [3]
of hydrogen gas:
i) Granulated zinc is preferred to metallic zinc.
ii) Dilute nitric acid is not used for the preparation of hydrogen.
iii) Copper is not used with dilute acids to prepare hydrogen.

B. Electronic configuration of an element ‘T’ is 2,8,7 : [3]

i) What is the group number of “T”?


ii) In which period does “T” belong?
iii) Mention its valency.

5
C. Solve the following numerical: [4]
i) A gas is allowed to expand at a constant temperature from an initial volume of
300 ml. to a final volume of 1800 ml. At the end of the expansion; the pressure
of the gas was found to be 1 atmosphere. What was the initial pressure of the
gas?
ii) A given mass of gas occupies a volume of 200 ml. at 37oC. To what
temperature must the gas be heated to make its final volume as 500 ml?
Assume that the pressure of the gas remains constant.

Question 5.
. A. Complete the following table : [4]
Common name Chemical name Chemical Efflorescent/
formula Hygroscopic/
Deliquescent
i) Glauber’s salt Sodium sulphate
decahydrate ______________ ____________
ii) Washing soda
_____________ Na2CO3.10H2O ____________

B. Given below is an atomic orbital diagram of a compound. Study the


diagram and answer the following questions:
[4]

`
i) State the type of bond formed in the compound.
ii) How many lone pairs are there in the compound?
iii) State the number of bonds formed in the compound.
iv) How many electron pairs are being shared by oxygen atom?

6
C. Select the correct formula from the bracket for each of the following [2]
compound:
i) Potassium chlorate [KClO3 / KClO4]
ii) Mercury(l) oxide [ Hg2O / HgO ]

Question 6.
A. Give balanced chemical equations for the following reactions: [4]
i) Calcium oxide reacts with carbon dioxide to give calcium carbonate.
ii) Sodium hydroxide solution reacts with zinc sulphate solution to give sodium
sulphate and zinc hydroxide.
iii) Magnesium reacts with copper (ll) sulphate solution to give magnesium
sulphate and metallic copper.
iv) Chlorine reacts with potassium bromide to form potassium chloride and
bromine.

B. An element belongs to the third period and group two of the periodic [3]
table.; then state the following:
i) Valency of the element.
ii) Name of the element.
iii) The formula of its chloride.

C. Identify the following: [3]


i) The law that explains resemblance between the properties of the first and
eighth element when they are arranged in increasing order of their atomic
weights.
ii) An element which does not contain any neutron in its nucleus.
iii) A desiccating agent.

Question 7.
A. Name the following: [4]
i) The temperature -273oC is called_________.
ii) The gas law that explains relationship between pressure and volume of a gas
at constant temperature.

7
iii) The standard pressure of a gas in cm. of mercury corresponding to one
atmospheric pressure.
iv) The absolute temperature value corresponding to 35oC.

B. When Sodium sulphite reacts with dilute Sulphuric acid; [4]


i) Name the gas evolved.
ii) Write balanced equation for the above equation.
iii) Identify the product obtained when the gas evolved in the above reaction is
treated with water.
iv) Which type of chemical reaction is taking place in the above reaction with
water?

C. Define the following [2]


i) Amphoteric oxides
ii) Reducing agent

*****

8
VIBGYOR HIGH
Sample Paper
COMMERCIAL STUDIES
Grade: IX Max. Marks : 80
Date: Time Allowed: 2 hours

INSTRUCTIONS:
 Answers to this paper must be written on the paper provided separately.
 You will not be allowed to write during the first 15 minutes.
 This time is to be spent in reading the question paper.
 The time given at the head of this paper is the time allowed for writing
the answers.
 The intended marks for the questions or parts of questions are given
alongside the questions.
 This question paper consists of 4 pages.
______________________________________________________________________

Section A (40 marks)


Attempt all questions from this section

[10]
Question 1
Distinguish between:
(a) Domestic trade and Foreign trade [2]
(b) Joint Stock Company and One Person Company [2]
(c) Marketing and Selling [2]
(d) Profit Organisation and Non-Profit Organisation [2]
(e) Book-Keeping and Accounting [2]

Question 2 [10]
(a) What is ‘Business’? [2]
(b) Explain the principle of ‘one man one vote’. [2]
(c) What are Aids to Trade? Give one example. [2]
(d) What is Production Management? [2]
(e) What is Recruitment? [2]

1
Question 3 [10]
(a) Define the Term ‘Production’. [2]
(b) What is an Accounting Cycle? [2]
(c) Define the Term ‘Profession’. [2]
(d) What is the modern approach to marketing? Explain with the [2]
help of an example.
(e) Write a note on ‘Current Deposit Account’. [2]

Question 4 [10]
(a) Briefly explain any two characteristics of Business Activities. [2]
(b) What is Limited Liability Partnership? [2]
(c) What is Business Finance? [2]
(d) State and explain any two objectives of Human Resource [2]
Department.
(e) What is meant by Non-Commercial Activities? Give one [2]
example.

SECTION B (40 marks)


(Attempt any four questions)
Question 5 [10]
(a) What is Non-profit organisation? Explain the types of Non-profit [5]
organisation.
(b) “The One-Man control is the best”. Support your answer by [5]
giving any five features of Sole Proprietorship.

Question 6 [10]
(a) Define Commercial Organisations? Explain any three features [5]
of Private Sector Enterprises+.
(b) What do you understand by Financial Management? Give its [5]
objectives.

2
Question 7 [10]
(a) “A Joint Stock Company is the best form of business [5]
organization”. Justify your answer by providing any five features
of it.
(b) Explain the following terms: [5]
(i) Liability
(ii) Asset

Question 8 [10]
(a) What are Joint Sector Enterprises? State its features. [5]
(b) Explain any five Agency functions of Commercial Bank. [5]

Question 9 [10]
(a) Discuss any five features of Partnership. [5]

(b) ‘Cooperative Societies serves as social utility to community’. [5]


Explain any five features to support your answer.

Question 10 [10]
Prepare a single column cash book from the following
transactions of Mukesh Agarwal for the month of January 2020.

PTO…

3
2020 Amount

Jan, 1 Cash in hand 4,510


Jan, 5 Purchase goods from Mr. Roy in 15,000
cash
Jan, 7 Received Commission in cash 3,500
Jan, 9 Sold goods in cash 25,000
Jan, 11 Purchased furniture by cheque 20,000
Jan, 13 Purchased furniture by cash 10,000
Jan, 15 Paid electricity bill 3,800
Jan, 17 Paid staff salary 5,200
Jan, 19 Withdrawn cash from business for 5,000
personal use
Jan, 20 Cash further invested in business 20,000
Jan, 21 A cheque received from Mr. Roy 15,000
and deposited into bank
Jan, 23 Purchased machinery in cash 10,000
Jan, 25 Received Cash from Anand 5,000
Jan, 27 Cash Sales for the third week 20,000
Jan, 29 Paid Rent 1,500
Jan, 29 Sold news paper 100
Jan, 30 Paid Commission in cash 500
Jan, 31 Paid wages 400

*****

4
VIBGYOR HIGH
Sample Paper
COMPUTER APPLICATIONS

Grade: IX Max. Marks: 100


Date: Time Allowed: 2 hours

INSTRUCTIONS:
 Answers to this paper must be written on the paper provided separately.
 You will not be allowed to write during the first 15 minutes.
 This time is to be spent in reading the question paper.
 The time given at the head of this paper is the time allowed for writing the answers.
 The intended marks for the questions or parts of questions are given alongside the
questions.
 This paper is divided into two Sections
 Attempt all questions from Section A and any four questions from Section B.
 This question paper contains 6 printed pages.

SECTION A (40 marks)


Attempt all questions
Question 1 [10]
(a) What is Inheritance? Explain with an example. [2]
(b) Write a Java statement to create an object Rectangle from the class [2]
Shapes. Also mention any two attributes of the object “Rectangle”.
(c) What are the default values of primitive data type char and boolean? [2]
(d) Rewrite the following code snippet using ternary operator: [2]
if(n>0)
System.out.println("Positive");
else{
if(n<0)
System.out.println("Negative");
else
System.out.println("Zero"); }

1
(e) What is the difference between Scanner class functions next( ) and [2]
nextLine( )?

Question 2 [10]
(a) What is the value of k after evaluating the following expression? [2]
k-=k++ * 2 - (k-- / 2) + k ; when int k=16 initially
(b) Give the output of the following program segment. [Show the dry run] [2]
for(int m=5; m<=20; m+=5)
{
if(m%3==0)
break;
else
if(m%5==0)
System.out.println(m);
continue;
}
(c) Write a Java statement to print the following sentence: [2]
“Be the CHANGE that you wish to see in the world”
MAHATMA GANDHI
(d) Give the output of the code snippet given below: [2]
char ch='1';
switch(ch)
{
case '1':
System.out.println("Java");
default:
System.out.println("C++");
case '2':
System.out.println("Python");
break;
case '3':
System.out.println("Ruby");
break;
}

2
(e) Define the term byte code. [2]

Question 3 [20]

(a) Convert the following while loop to corresponding for loop: [2]
int m=5, n=10;
while(n>=1)
{
System.out.println(m*n);
m*=2;
n--;
}
(b) Why class is called an object factory? [2]
(c) What will be the output of the code snippet given below if num=1: [2]
int x=0 , y=0;
if(num>0)
{
System.out.println(x=x++);
System.out.println(y=y++);
}
(d) Arrange the following operators from higher precedence to lower [2]
precedence
&& , !=, ++ , %
(e) Write a Java expression for the following: [2]
by10 −5
res=
b5
(f) State one similarity and one difference between for loop and while loop. [2]
(g) Give the output of the following code snippet. [Show the steps] [2]
char i=’Z’;
System.out.println((char) (i-‘A’+’a’)+’1’);
(h) Write the output and name the type of conversion used in the following [2]
statement:
System.out.println((float)5.399);
(i) Name the type of error (compile time, runtime or no error) in each case [2]
given below:

3
i. System.out.println(double x=2);
ii. int z= 6 * 4 / 3 * 0;
(j) Write the output of the following code snippet: [2]
double n= - 8.99;
System.out.println(Math.round(n++) - Math.ceil(n));

SECTION B (60 marks)


Attempt any four questions from this Section
The answers in this Section should consist of the Programs in either Blue
J environment or any program environment with Java as the base.
Each program should be written using Variable description/Mnemonic
Codes so that the logic of the program is clearly depicted.
Flowcharts and Algorithms are not required.

Question 4 [15]

GoFast taxi service charges to the passenger’s as per the following


conditions:
Kilometers travelled (km) Rate/km (Rs)
≤ 2 km 25
2< km ≤ 6 10
6< km ≤ 12 15
12< km ≤ 18 20
>18 km 25
Write a program to accept the kilometers travelled. Calculate and display
the total amount to be paid by the passenger.

Question 5 [15]

Write a program to find and display the sum of the series given below:
S= 2x/2 – 4x/5 + 6x/8 – 8x/11 + …………. n terms

4
Question 6 [15]
Using switch statement, write a menu driven program to calculate the
maturity amount of a Bank Deposit based on the following options given to
the user:
(i) Term Deposit: Calculate and print the maturity amount (A) receivable using
the formula
r
A=P[1+ ]n , where P is Principal, r is rate of interest and n is time
100
period (in years).

(ii) Recurring Deposit: Calculate and print the maturity amount (A) receivable
using the formula
n(n+1) 𝑟 1
A = (P * n) + (P * * * ) , where P is Monthly installment, r is
2 100 12
rate of interest and n is time period (in months)
For an incorrect option, an appropriate message should be displayed.

Question 7 [15]
Write a program to accept a number and check and display whether it is an
Emirp number or not.
An Emirp number is a number which is prime backwards and forwards.
For example: 13 is an Emirp number since 13 and 31 are both prime
numbers.

Question 8 [15]
An ABC Mall has announced the festive discounts on the purchase of items
based on the total cost of items as per the given criteria:

Total cost Home Furniture


Appliances
Less than 25000 5% 7%

25001 - 30000 7% 10%


30001 – 45000 10% 13%
45001 and above 14% 16%

5
Write a program to input the name of the customer, total cost of purchase,
type of purchase (H for Home Appliances and F for Furniture). Compute
and display the amount to be paid by the customer after availing the
discount in the following format:
Name Cost of purchase Discount (in Rs) Net amount to be paid
****** ***** ***** *****
Question 9 [15]
Write a program using switch statement to perform the following:
(i) To display the following series:
1, 2, 5, 12, ………………………….. 503
(ii) To print the value of Y
Y= 3x2 + 2x, where x ranges from -5 to 5 with an increment of 2

*****

6
VIBGYOR HIGH
Sample Paper
ENGLISH LANGUAGE
Grade: IX Max. Marks: 80
Date: Time Allowed: 2 hour

INSTRUCTIONS:

 Answers to this paper must be written on the paper provided separately.


 You will not be allowed to write during the first 15 minutes.
 This time is to be spent in reading the question paper.
 The time given at the head of this paper is the time allowed for writing the
answers.
 The intended marks for the questions or parts of questions are given
alongside the questions.
 Attempt all five questions.
 This question paper contains 7 printed pages.
_____________________________________________________________________________________________________________________________ __________

Question 1 (Do not spend more than 30 minutes on this question.) [20]

Write a composition (300 – 350 words) on any one of the following:

(a) Write an original short story ending with the words:

Never again will I make the same mistake and fall into such a nasty
situation.

(b) ‘School life is the best phase of an individual’s life.’


Discuss for or against the topic.
(c) Write an account of an event which has happened in the last year and
also provide the details of how it has affected the destiny of many
people in your country.

1
(d) You have gone to the railway station to see your friend off. You have
plenty of time on hand when you reach there. What do you do to while
away your time? What meets your eyes? Write a description of the
station and its normal activities.

(e) Study the picture given below. Write a short story or description or an
account of what the picture suggests to you. Your composition may
be about the subject of the picture or you may take suggestions from
it. However, there must be a clear connection between the picture and
the composition.

Question 2 (Do not spend more than 20 minutes on this question.) [10]

Select one of the following:


(a) You had been for a cycling expedition near the Jim Corbett National
Park. Write a letter to your younger sister describing your
experience of cycling amidst the sights, sounds and smell of nature
for the first time.

(b) Write a letter to the Principal of your school suggesting the introduction
of gardening as a hobby class in your school.

2
Question 3 (a) Your school is organizing an educational trip to Pachmarhi in Madhya [5]
Pradesh. Write a notice to be put up in your school informing the
students about the trip.

(b) Write an email to the manager of a travel agency, requesting him/her [5]
to provide details about a tour package to Pachmarhi, for a group of
hundred students of your school.

Question 4 Read the passage and answer the questions that follow: [20]
James was not indifferent to the wishes of his family that he should
get married to a nice girl. Besides, when his uncle departed this life
and bequeathed him a comfortable little legacy, it really seemed the
correct thing to do to set about discovering someone to share it with
him. He thought of Joan Sebastale as the most suitable young
woman to whom he might propose marriage. James walked across
the park towards the Sebastale residence in a positive frame of
mind.

His thoughts were interrupted by a sound of clock striking the half-


hour. Half -past four. A frown of dissatisfaction settled on his face.
He would arrive at the Sebastale mansion just at the hour of
afternoon tea. Joan would be seated at a low table, spread with an
array of silver kettles and cream-jugs and delicate porcelain
teacups. Behind the table, her voice would tinkle pleasantly in a
series of little friendly questions about weak or strong tea; how
much, if any, sugar, milk, cream and so forth: “Is it one lump? I
forgot. You do take milk, don’t you? Would you like some more hot
water, if it’s too strong?” James detested the questioning pattern in
the whole system of the afternoon tea.

Now, as he passed through small streets that led to Joan’s


residence, a horror at the idea of confronting Joan at her tea table
seized on him. But suddenly an idea came to his mind. On one floor
of a narrow little house at the end of the street lived Rhoda Ellam, a

3
remote cousin. James decided to visit her and delay by half-an-hour
the important business of the marriage proposal to Joan.

Rhoda welcomed him to her house. “I’m having a high tea,” she
announced. “Find yourself a cup; the teapot is behind you. Now tell
me about hundreds of things.” She talked amusingly and made her
visitor talk amusingly too. James found that he was enjoying an
excellent tea without having to answer routine questions about the
strength of the tea and amount of sugar in it.

“And now tell me why you have come to see me,” said Rhoda
suddenly.

“There is nothing special,” said James. “I was passing by your house


and I just thought I’d look in and see you. Since I’ve been talking to
you, however, a rather important idea has occurred to me…”

Some forty minutes later James returned to his family, bearing an


important news. “I am engaged to be married,” he announced. On a
September afternoon of the same year, after the James-Rhoda
wedding celebrations had ended, James came in to the drawing-
room of his house. Rhoda was seated at a low table, behind dainty
porcelain cups and gleaming silver. There was a pleasant tinkling
note in her voice as she handed him a cup, saying; “You like it
weaker than that, don’t you? Shall I put some more hot water to it?
No?”

(a) Give the meaning of the following words as used in the [3]
passage. One-word answers or short phrases will be accepted.

(i) bequeathed
(ii) array
(iii) detested

4
(b) Answer the following questions briefly in your own words.

(i) Give one reason to show that James wished to marry. [1]
(ii) Why did James decide to go to Joan’s residence? What does the [2]
phrase, ‘in a positive state of mind’ indicate?
(iii) What and why did James dislike in the pattern of serving tea in the [2]
afternoon?
(iv) Who was Rhoda Ellam? Why did James decide to visit her? [2]
(v) “…….. a rather important idea has occurred to me.” What was the [2]
important idea?
(c) In not more than 50 words, show how fate made James face the [8]
awkward situation at the end, which he had avoided earlier in the
story.

Question 5 [20]
(a) Fill in each of the numbered blanks with the correct form of the [4]
word given in brackets. Do not copy the passage, but write in
correct serial order the word or phrase appropriate to the blank
space.

Example: (0) claimed

Harish, a young taxi driver_____0_____(claim) that a monster had


____1___(attack) him while he was ___2___(drive) along a lonely
road. The monster _____3____ (be) five feet tall, had red eyes and a
hairy body. After ____4______(chase) the taxi for a kilometer, he had
jumped on its roof and scratched the face of the driver. Many people
said that they believed in the _____5____ (exist) of the monster and
the newspaper called the monster the Monkey-man. Ninety
policemen set out to ____6_____ (catch) the monster and the local
television offered Rupees ten thousand to anyone who would get him
dead or alive. The Monkey-man _______7_____(become) so popular
that crowds came to Delhi to see him. No one has _____8_____ (find)
him. As everyone knows, monsters may or may not exist, but they are
good for the tourist industry.

5
(b) Fill in the blanks with appropriate words: [4]

(i) Can we count ____________ him to help you?

(ii) The novel has been translated ________ many languages.

(iii) The new Government has brought _______ many changes.

(iv) I’ll find you a good contractor to carry __________ the work.

(v) The shirt doesn’t go ________ that suit.

(vi) Who will look _________ our dog while we are out?

(vii) He puts ________ some money every month.

(viii) Her age is beginning to tell ________ her.

(c) Join the following sentences to make one complete sentence [4]
without using ‘and’, ‘but’ or ‘so’.

(i) His illness was serious. He stayed in bed for one month.

(ii) The project must be completed. It does not matter what it costs.

(iii) I must know the facts. I cannot help you otherwise.

(iv) Smita is a good tennis player. There is no doubt about it.

(d) Rewrite the following sentences according to the instructions [8]


given after each. Make other changes that may be necessary, but
do not change the meaning of each sentence.

(i) Prerna is not the cleverest girl in the class.


(Begin: Some…)
(ii) We did not have a good time at the picnic because it was too hot.
(Begin: If…)
(iii) Of what use is this dress to you?
(End: … you.)

6
(iv) She is terrified at the thought of going out alone.
(Use: ‘terrifies’)
(v) Put your tools away, people may fall over them.
(Use: ‘lest’)
(vi) She said, “If you cannot face the music, I will have to.”
(Begin: She told him that …….)

(vii) Although she was too young, they appointed her.

(Begin: In spite….)

(viii) Nobody will deny that she is honest.

(Begin: Everyone ……)

*****

7
VIBGYOR HIGH
Sample Paper
ENGLISH LITERATURE
Grade: IX Max. Marks: 80
Date: Time Allowed: 2 hour
INSTRUCTIONS: -
 Answers to this paper must be written on the paper provided separately.
 You will not be allowed to write during the first 15 minutes.
 This time is to be spent in reading the question paper.
 The time given at the head of this paper is the time allowed for writing the
answers.
 Attempt five questions in all.
 You must attempt at least one question from each of the Sections A, B and C
and not more than two other questions from the same books you have already
compulsorily chosen.
 The intended marks for questions or parts of questions are given in brackets [ ].
 This question paper contains 6 printed pages.
_______________________________________________________________________
Section A – Drama
The Merchant of Venice: Shakespeare
Question 1 [16]
Read the extract given below and answer the questions that follow:
Bassanio:
To you, Antonio,
I owe the most, in money and in love;
And from your love I have a warranty
To unburden all my plots and purposes
How to get clear of all the debts I owe.

Antonio:
I pray you, good Bassanio, let me know it;

(i) Describe Antonio’s mood at the beginning of the scene. State any two [3]
reasons that Antonio’s friends, who were present gave to explain his

1
mood.
(ii) What promise did Antonio make to Bassanio immediately after this? [3]
(iii) What did Bassanio say to Antonio about ‘a lady richly left’ in Belmont? [3]
(iv) Why was Antonio unable to lend Bassanio the money that he [3]
needed? How did he propose to help his friend?
(v) What does the above extract reveal of the relationship between [4]
Antonio and Bassanio?

Question 2 [16]
Read the extract given below and answer the questions that follow:
Therefore, I part with him; and part with him
To one that I would have him help to waste
His borrow’d purse.

(i) Naming the speaker, state who he is parting with. Why is the speaker [3]
happy to part?
(ii) What is the ‘borrow’d purse’? Why had the speaker decided to accept [3]
the invite?
(iii) What instructions did the speaker give before leaving and to whom? [3]
(iv) What does the proverb ‘Fast bind, Fast find’ mean? What did the
speaker think of the person he was parting with? What did the speaker
call him? [3]
(v) Shylock’s passion for revenge overpowers his love for his daughter. [4]
Justify the above statement providing apt references from the play ‘The
Merchant of Venice’.

Question 3 [16]
Read the extract given below and answer the questions that follow:
Why, I am sure, if he forfeit, thou wilt not
take his flesh: what’s that good for?

(i) Where does the scene take place? Who is the speaker here and whom [3]
is he addressing?
(ii) Whose ‘flesh’ does the speaker refer to here? How is the question [3]
raised in the extract, answered then?

2
(iii) There I have another bad match: [3]
Whom does the speaker consider to be ‘another bad match’ and why?
Who was the previous ‘bad match’ and why?
(iv) The bond is an integral part of the play. Discuss the bond in detail. [3]
(v) Mentioning any six accusations made against ‘he’, describe how it had [4]
strengthened the resolve of ‘thou’.

Section B – Poetry
Treasure Trove: A Collection of ICSE Poems and Short Stories

Question 4 [16]
Read the extract given below and answer the questions that follow:
The most important thing we’ve learned,
So far as children are concerned,
Is never, NEVER, NEVER let
Them near your television set.
(Television - Roald Dahl)

(i) To whom is the poem addressed? What has the poet noticed in almost [3]
every house he has been?
(ii) Why do the parents allow their children to watch television? [3]
(iii) What is the negative impact of watching television on tender minds? [3]
Mention any six.
(iv) Name the English writer whose books the ‘younger ones’ read. Where [3]
were the books found?
(v) What did the poet request the parents to do? How would the children [4]
react initially when the parents followed the advice? What change would
be noticed in them later?

PTO…

3
Question 5 [16]
Read the extract given below and answer the questions that follow:
“Now tell us what ‘twas all about,”
Young Peterkin, he cries;
And little Wilhelmine looks up,
With wonder – waiting eyes;
“Now tell us all about the war,
And what they fought each other for.”
(After Blenheim - Robert Southey)
(i) Describe the scene at the beginning of the poem. [3]
(ii) What did Young Peterkin find and where? Describe it. [3]
(iii) Who is referred to as ‘each other’? Who was praised by everybody and [3]
why?
(iv) How was Kaspar’s family affected by the war? [3]
(v) Mentioning any two consequences of such ‘great and famous’ victories, [4]
state the message that the poet wishes to convey to the readers. How
has the poet successfully brought out the generation gap in the
reactions to the war?

Question 6 [16]
Read the extract given below and answer the questions that follow:
For oft, when on my couch I lie
In vacant or in pensive mood,
They flash upon that inward eye
Which is the bliss of solitude;
(Daffodils - William Wordsworth)
(i) What flashes upon the poet when he is in a pensive mood? What is its [3]
effect on the poet?
(ii) Where does the poet come across the daffodils? [3]
(iii) What are the daffodils compared to? Why? [3]
(iv) With whom do the daffodils compete and with what result? Why is the [3]
poem considered lyrical?
(v) How does the poet personify the daffodils? Describing the poet’s state [4]
of mind in the beginning of the poem state the effects that nature has
on man.

4
Section C – Prose
Treasure Trove: A Collection of ICSE Poems and Short Stories
Question 7 [16]
Read the extract given below and answer the questions that follow:
Idleness, vice and intemperance had done their miserable work,
and the dead mother lay cold and still amid her wretched children.

(i) What help did the villagers provide to the orphaned children? [3]
(ii) What is the ‘poor house’ and who suggested that Maggie should be sent [3]
there? Why was the suggestion agreed upon by everyone present?
(iii) What kind of a lady was Joe Thompson’s wife and why? [3]
(iv) What did Mr. Thompson see through the little chamber window when [3]
he returned from work? What did he understand of the expressions on
the little girl’s face?
(v) Who was considered a ‘precious burden’ and why? How did this [4]
‘precious burden’ transform the Thompson household?

Question 8 [16]
Read the extract given below and answer the questions that follow:
It had no eyes, ears, nose or mouth. It was just a round smooth
head - with a school cap on top of it! And that’s where the story
should end. But for Mr. Oliver it did not end here.
(i) Who was Mr. Oliver? Where and when did he encounter ‘it’? [3]
(ii) Where did Mr. Oliver work? Why did Life magazine call it the ‘Eton of [3]
the East’?
(iii) Why did Mr. Oliver stop on his way? Why did he move closer? Why did [3]
his anger give way to concern?
(iv) What did Mr. Oliver do after ‘it’ looked up? How was his reaction [3]
contrary to what had been earlier said about him?
(v) What were the questions raised by Mr. Oliver that show that he was a [4]
concerned man? Describing the end, state what provides the elements
of suspense in this story.

5
Question 9 [16]
Answer the following questions with reference to Ernest Hemingway’s
short story ‘Old man at the Bridge’.
(i) Describing the activities taking place on the pontoon bridge, state the
tasks that were assigned to the narrator of the story. [4]
(ii) Describing the old man, state from where he had come. Why was he [4]
feeling guilty sitting on the bridge?
(iii) Hemingway uses the old man at the bridge to criticise war. [8]
Bringing out the hopelessness and the helplessness of the old man
state the reason why there is a missing article in the title of the story.
Elaborate upon the ironical end and state what was the only luck in
store for the old man.

*****

6
VIBGYOR HIGH
Sample Paper
French
Grade: IX Max. Marks : 80
Date: Time Allowed: 3 hours

INSTRUCTIONS:

 Answers to this paper must be written on the paper provided separately.


 You will not be allowed to write during the first 15 minutes.
 This time is to be spent in reading the question paper.
 The time given at the head of this paper is the time allowed for writing the answers.
 The intended marks for the questions or parts of questions are given alongside the
questions.
 This question paper contains 5 printed pages.

Répondez à toutes les questions

Question 1 Écrivez en français une rédaction d’environ 200 mots sur [20]
un des sujets suivants:

(a) L’importance de loisirs.


(b) Racontez une journée extraordinaire que vous avez
passée à la plage avec vos amis.
(c) Continuez cette histoire : «Hier soir, après l‛école, je suis
arrivé(e) à la maison. Il n’y avait personne. La porte était
fermée à clé et je ne pouvais pas entrer…».
(d) L’importance de coutumes.
(e) Faites une description de l’image donnée ci-dessous.

1
Question 2 [15]
Écrivez en français une lettre sur un des sujets suivants :
(a) Écrivez une lettre à votre ami français(e) dans laquelle
vous décrivez votre expérience de garder votre petit cousin /
petite cousine.
OU
(b) Vous avez passé quelques jours chez un/e
ami /e. Écrivez-lui une lettre en lui remerciant. Parlez des
choses que vous avez aimées pendant votre séjour.

Question 3 Lisez l’extrait suivant et répondez aux questions qui [10]


suivent sans répéter les mots du texte :

L'Himalaya est un ensemble de montagnes magnifiques.


Mais si l'Himalaya est très connu c'est grâce à une de ses
montagnes, l'Everest. L'Everest est la plus haute montagne
du monde. On lui a donné le nom de 'Toit du Monde'. Tous
les alpinistes rêvent de grimper à son sommet mais très peu
y arrivent.

Temba Tsheri est né au pied de l'énorme montagne. Son


père, comme son grand-père, était 'Sherpas', c'est-à-dire
guides de l'Himalaya pour les alpinistes. Les Sherpas sont
des accompagnateurs précieux pour les alpinistes parce
qu'ils sont habitués à vivre très haut en altitude. Ils peuvent
grimper l'Everest sans bouteilles d'oxygène. Pour gagner leur
vie, les Sherpas guident les alpinistes et portent souvent leurs
bagages pendant la montée.

Mais le rêve de Temba, c'est de devenir le plus jeune


alpiniste à arriver au sommet de l'Everest. Il se met en route
en avril. Il veut arriver au sommet le 18 mai 2000. Mais ce
2
jour-là, alors qu'il est à seulement 20 mètres du sommet, une
terrible tempête se lève. Temba doit abandonner et
redescendre: Un an plus tard, Temba se remet en route.
Cette fois, il emporte des bouteilles d’oxygène pour l’aider à
respirer.

Le 22 mai 2001, il arrive au sommet. Temba a gagné : il est,


de loin, le plus jeune alpiniste à être arrivé sur le ‘Toit du
Monde’. Une vraie aventure, mais le gouvernement népalais
ne veut pas que d’autres adolescents, encore plus jeunes
que Temba, risquent leur vie. Il a interdit aux moins de 16 ans
l’ascension de l’Everest.

(a) Pourquoi les Sherpas sont très importants pour les

alpinistes ? [2]
(b) Qu’est-ce que c’est le rêve de Temba et comment était le
premier trajet de Temba vers son rêve, a-t-il réussi ? [2]
(c) Décrivez le deuxième trajet de Temba vers son rêve, a-t-il
réussi ? [2]
(d) Quelle étape prend-il le gouvernement népalais après le
succès de Temba ? [2]
(e) Qu’est-ce que c’est le toit du monde ? [2]

Question 4 (a) Mettez les verbes entre parenthèses à la forme qui


convient :
[5]
Les étudiants _______________ (aller) au restaurant avec
(i)
leur professeur demain.
____________ (parler) lentement ! Je ne vous comprends
(ii)
pas.
La semaine dernière, nous ____________ (passer) deux
(iii)
jours à Paris.
Pendant mon enfance, je __________ (visiter) mes grands -
(iv)
parents chaque année.
Si tu gagnais à la loterie, tu ____________ (acheter) une
(v)
maison.

3
(b) Remplacez les mots soulignés par des pronoms
convenables : [4]
(i) Le professeur a expliqué la leçon à ses élèves.

(ii) Il donne des fleurs à son amie.

(iii) Nous devons aller à la salle à manger.

(iv) Je prendrai du pain au petit- déjeuner.

(c) Transformez les phrases suivantes avec le comparatif


ou le superlatif : [2]

(i) Cette maison est (bon) que cette villa.

(ii) Le gâteau au chocolat est (délicieux) des gâteaux servis sur


la table.

(d) Transformez les phrases suivantes en utilisant les


expressions de négation : [2]

(i) Tu travailles encore le samedi.

(ii) Ma voisine prend toujours ma voiture.

(e) Reliez les phrases en employant des pronoms relatifs : [2]


(i) Eugène a rencontré un français au café. La mère du français
est indienne.
(ii) Connais – tu cet homme ? Il veut aller à la lune.

Question 5 (a) Traduisez le texte suivant en anglais : [5]

On dit que la danse, c’est seulement pour les filles. Moi,


aussi, je pensais ça, mais ce n’est pas vrai ! Je fais de la
danse moderne deux fois par semaine et, le week-end, il y a
quelquefois des concours. C’est fatigant, c’est une vraie
activité sportive, c’est amusant et en plus, on est entouré de
copains !

4
(b) Traduisez le texte suivant en français :
[5]
I spent a very interesting afternoon at the Louvre - normally I
don't like museums that much! But I entered around 2 pm
through the Pyramid and was in a group with my teacher who
knows a lot about art. We came out finally around 6 pm and I
even bought a book on the artwork of The Louvre!

(c) Vous êtes en France pour la première fois et vous [10]


devez aller à l’office du tourisme mais vous ne
connaissez pas le chemin. Écrivez un dialogue entre
vous et un homme / une femme près de vous pour
demander les renseignements.
(Salutations - le chemin - les directions - le numéro d’autobus
- remercier)

*****

5
VIBGYOR HIGH
Sample Paper

GEOGRAPHY
Grade: IX Max. Marks: 80
Date: Time Allowed: 2 hours

INSTRUCTIONS:

 Answers to this paper must be written on the paper provided separately.


 You will not be allowed to write during the first 15 minutes.
 This time is to be spent in reading the question paper.
 The time given at the head of this paper is the time allowed for writing the answers.
 The intended marks for the questions or parts of questions are given alongside the questions.
 This question paper contains 8 printed pages.
.

 Attempt seven questions in all.


 Part I is compulsory. All questions from Part I are to be attempted.
 A total of five questions are to be attempted from Part II.
 The intended marks for questions or parts of questions are given in brackets [ ].

Note :

(i) In all Map Work, make wise use of arrows to avoid overcrowding of the map.
(ii) The Map given at the end of this question paper must be attached,to your answer booklet.

(iii) All sub-sections of the questions attempted must be answered in the correct
serial order.
(iv) All working including rough work should be done on the same answer sheet which is used to
answer the rest of the paper.

1
PART I (30 Marks)

(Attempt all questions from this Part)

Question 1 Answer the following questions in brief: [20]

(a) How many time zones has the earth been divided into? How many [2]
longitudes does each time zone cover?

(b) Name the two chief motions of the earth? [2]

(c) What is Gutenberg Discontinuity? At what depth does it begin? [2]

(d) (i) Why are sedimentary rocks called stratified rocks? [2]

(ii) What is lithification?

(e) What are active volcanoes? Give an example of an active volcano. [2]

(f) Name the instrument used to measure: [2]

(i) The direction of the movement of earthquake waves.

(ii) The intensity of amount of energy released during an earthquake.

(g) (i) What are tides? [2]

(ii) Name the tides caused when the earth, moon and sun are in the
same line.

(h) (i) Name the layer above the tropopause. [2]

(ii) What is its extent above sea level?

(i) Draw a well labelled diagram of Orographic or Relief rainfall. [2]

(j) Name two factors favouring evaporation. [2]

Question. 2 Mark and label the following on the outline map of the world: [10]

(a) North Sea

(b) South Atlantic Ocean

(c) River Orange

(d) Tropical Monsoon Region in South-East Asia

2
(e) Strait of Gibraltar

(f) Gulf of Carpentaria

(g) Rockies

(h) Andes

(i) Tibetan Plateau

(j) Hot Desert Region in Australia

PART II (50 marks)


(Attempt any five questions from this Part)

Question.3

(a) Name the two main latitudes in the Southern Hemisphere with the [2]
degrees.

(b) Which are the two longitudes which divide the earth into Western [2]
and Eastern Hemispheres?

(c) Give a geographical reason for the following: [3]

(i) Lines of longitudes are called Meridians.

(ii) The earth is not a perfect sphere in shape.

(iii) Norway is called the ‘Land of Midnight Sun’.

(d) With respect to the position of the sun on 22nd December, [3]
answer the following:

(i) Which hemisphere is tilted towards the sun?

(ii) What is the season in the Northern and Southern Hemisphere


respectively?

(iii) What is the duration of day in the polar region of that hemisphere?

Question. 4

(a) Which is the outermost layer of the earth? What is its thickness [2]
above mountains and oceans respectively?

3
(b) Mention two factors which help in acquiring knowledge of the [2]

interior of the earth?

(c) Answer in brief: [3]

(i) The inner core is in a solid state. Explain why?

(ii) Give two points of distinction between Acid and Basic Igneous
Rocks.
(d) Identify the type of rock: [3]

(i) Peat

(ii) Graphite

(iii) Basalt

Question 5
(a) Name the following processes of weathering: [2]

(i) Weathering caused by reaction of minerals in rocks with oxygen.

(ii) Weathering caused by expansion of minerals in rocks coming into


contact with rainwater.
(b) State any two types of physical weathering. [2]

(c) With reference to the products of volcanic eruptions answer [3]


the following:

(i) Which is the most abundant gas that comes out?

(ii) What are solid fragments of rock particles known as?

(iii) What are the small stone sized particles known as?

(d) What are the three major earthquake belts of the world?. [3]

Question 6 (a) Name the winds which influence ocean currents in: [2]

(i) The Equatorial Region

(ii) The Temperate Latitudes


(b) What are surface currents? How much percentage of water do they [2]
constitute the ocean?

4
(c) Explain in brief how the following factors influence ocean [3]
currents:

(i) Salinity

(ii) Land

(iii) Earth’s rotation

(d) Give the geographical term for: [3]

(i) Large masses of water that circulate in regular patterns around


oceans.
(ii) The period when the earth and moon are at the farthest distance
from each other.

(iii) Tides formed when the earth, moon and sun are at right angles.

Question 7

(a) What is Global Warming? Give two causes of Global Warming. [2]

(b) What is Normal Lapse Rate? In which season is it more? [2]

(c) Give three characteristics of the Exosphere. [3]

(d) Draw and label the heat zones. [3]

Question 8
(a) What is atmospheric pressure? What is its unit of measurement? [2]

(b) Name any two local winds. [2]

(c) Distinguish between Tropical Cyclones and Temperate Cyclones.[3 [3]


points]

(d) Draw a neat and labelled diagram to show the permanent winds. [3]

Question 9
(a) State two types of radiation. [2]

5
(b) Define: [2]

(i) Evaporation

(ii) Humidity

(c) Name the region which gets 4 O’clock showers. Give the [3]
geographical reason for the same.

(d) Name three types of precipitation. [3]

Question 10

(a) Name the pollutants classified on the basis of: [2]

(i) Origin in environment

(ii) Degradation

(b) Mention four sources of noise pollution. [2]

(c) Define the following. Give an example of each. [3]

(i) Gaseous pollutant.

(ii) Particulate pollutant

(d) Explain the two major effects on human health caused by [3]
radioactive pollution.

Question 11

(a) What do you mean by ‘Natural Region’. [2]

(b) Within which latitudes do the Tropical Deserts lie? Name two [2]
places in Asia which fall within this natural region.

(c) With reference to Equatorial Region answer the following: [3]

(i) State its location.

(ii) The characteristic feature of its climate.

(iii) What type of season is experienced there?

6
(d) Mention the three distinct seasons experienced in the Indian sub - [3]
continent and the months in which they are experienced.

*****

7
8
VIBGYOR HIGH
SAMPLE PAPER

HINDI
Grade: IX Max. Marks: 80

Date: Time Allowed: 3 hours


___________________________________________________________________________________
Instructions: -
 Answers to this paper must be written on the paper provided separately.
 You will not be allowed to write during the first 15 minutes.
 This time is to be spent in reading the question paper.
 The time given at the head of this paper is the time allowed for writing the
answers.
 This paper comprises two sections - Section A and Section B.
 Attempt all questions from Section A.
 Attempt any four questions from Section B, answering at least one question each
from the two books you have studied and any two other questions.
 The intended marks for questions or parts of questions are given in brackets [ ].

SECTION A [40 MARKS]

Question 1 Write a short composition in Hindi of approximately 250 words on any [15]
one of the following topics:-

निम्िनिखित विषयों में से किसी एि विषय पर िगभग २५० शब्दों में संखिप्त िेि

निखिए :-

i. आज िा युग विज्ञाि िा युग है । ‘ विज्ञाि : िरदाि या अनभशाप ’ – इस विषय पर


अपिे विचारों िो तिकसंगत ढं ग से प्रस्तुत िीखजए |
ii. त्योहारों िा मािि जीिि में क्या महत्ि है ?मिुष्य िो उसिी संस्िृ नत से जोड़े रििे में

त्योहार किस प्रिार सहायि होते हैं ? अपिे विचार स्पष्ट िीखजए |
iii. िृि हमारे नमत्र हैं । प्रिृ नत िे संतुिि िो बिाए रििे में िृिों िी भूनमिा िो बताते

हुए िृि संरिण एिं संिर्कि िे विषय में अपिे विचार व्यक्त िीखजए |

1
iv. ‘दर्
ू िा दर्
ू पािी िा पािी’ इस उवक्त िो िेंद्र माििर िोई मौनिि िहािी निखिए |
v. िीचे कदए गए नचत्र िो ध्याि से दे खिए और नचत्र िो आर्ार बिािर िणकि िीखजए

अथिा िहािी निखिए खजसिा सीर्ा ि स्पष्ट संबंर् नचत्र से होिा चाकहए |

Question 2 Write a letter in Hindi in approximately 120 words on any one of the topics [7]
given below :-
निम्िनिखित में से किसी एि विषय पर कहं दी में िगभग १२० शब्दों में पत्र निखिए |

i. मोबाईि फ़ोि िरीदिे िी हठ िरिे िािे अपिे छोटे भाई/बहि िो मोबाईि फ़ोि िे
िाभ और हानियााँ बताते हुए एि पत्र निखिए |
अथिा
ii. िाहि चाििों िी असािर्ािी और यातायात िे नियमों िा पािि ि िरिे िे िारण

सड़ि पर होिे िािी दर्


ु टक िाओं िा िणकि िरते हुए किसी दै निि समाचार-पत्र िे
संपादि िो पत्र निखिए |

Question 3 Read the passage given below and answer in Hindi the questions that [10]
follow, using your own words as far as possible :-
निम्िनिखित गद्ांश िो ध्यािपूिि
क पकिए तथा उसिे िीचे नििे प्रश्नों िे उत्तर कहं दी
में निखिए | उत्तर यथासंभि आपिे अपिे शब्दों में होिे चाकहए |
योगासिों िा अभ्यास िरिे से शारीररि बि ही िहीं, मािनसि बि भी प्राप्त होता है ।
एिाग्रता बिती है ,िायक-दिता बिती है । इसमें संदेह िहीं कि आज मिुष्य िे
िाल्पनिि भौनति प्रगनत िर िी है । इससे िुछ हानियााँ भी हुई हैं । भीड़-भाड़,
प्रनतस्पर्ाक, िमता से अनर्ि िायक िरिा, शोरगुि आकद से तिाि और बेचि
ै ी बिी है |

2
तिाि अिेि शारीररि और मािनसि रोगों िो जन्म दे ता है | िई रोगों िा इिाज
पाश्चात्य नचकित्सा पद्धनतयााँ िहीं िर पा रही हैं ,किन्तु योगासिों से इि पर िाबू पाया
जा रहा है । यही िारण है कि आज बड़े -बड़े अस्पतािों में योग नचकित्सा विभाग
सफितापूिि
क िायक िर रहे हैं । योगासि किसी भी व्यवक्त िी शारीररि,मािनसि
िमताओं िा वििास िर उसे उसिे िाम में सफिता कदिाते हैं । उसिे मि िो
तिािमुक्त और प्रफुखल्ित रिते हैं | प्रारं भ में किसी िुशि प्रनशिि से योग सीििा
चाकहए | िह आपिी िमता और आिश्यिता िे अिुसार िुछ आसि नसिा दे गा ।
उििा र्ीरे – र्ीरे अभ्यास िरें । िुछ िोग आसिों िे अिेि िाभ पििर अपिे आप ही
आसि िरिे िगते हैं । ऐसा िरिा स्िास््य िे निए हानििारि हो सिता है ।
आजिि योगासि सीििे – नसिािे िे अिेि अिसर हैं । बहुत से विद्ाियों में योग
नशिि हैं । िुछ उद्ािों में प्रातःिाि योगासि नसिाए जाते हैं । यह व्यिस्था प्रायः
निःशुल्ि होती है | िुछ खजम, हे ल्थक्िब आकद संस्थाएाँ शुल्ि िेिर भी योगासि
नसिाती हैं | िुछ िोग अपिे र्रों में योगासि नसिाते हैं ।
बड़े ही हषक िा विषय है कि हमारे ितकमाि प्रर्ािमंत्री श्री िरे न्द्र मोदी जी िे प्रयासों से
21 जूि 2015 िो ‘अंतराकष्ट्रीय योग कदिस’ मिाया गया | इसमें विश्व िे िई दे शों िे
भाग निया ।
भारत िी प्राचीि योग विद्ा िो जि-जि ति पहुाँचािे एिं योग नशिा िे विषय में
जि जागृनत िरिे में स्िामी रामदे ि जी िा महत्िपूणक योगदाि है । पतंजनि योगपीठ
जैसे संस्थाि िी स्थापिा िर उन्होंिे ि िेिि िोगों िो योग िरिे िे निए प्रेररत किया
है , बखल्ि स्िदे शी िस्तुओं िा उत्पादि िरिे दे श िे वििास में भी महत्िपूणक भूनमिा
निभाई है । यकद हम भी अपिे ति-मि से योग अभ्यास िो अपिािर अपिे
जीिि िो स्िस्थ,सुन्दर और सफि बिाएाँगे तो यह दे श िे सुिहरे भविष्य िे निए एि सुिद
संिेत होगा ।

i. योगासि िरिे िे क्या िाभ हैं ? [2]


ii. मिुष्य िी प्रगनत िे साथ-साथ क्या हानियााँ हुई हैं ? [2]
iii. बड़े -बड़े अस्पतािों में योग नचकित्सा विभाग क्यों सफितापूिि
क िायक िर रहे हैं ? [2]
iv. आजिि योगासि सीििे – नसिािे िे िौि से अिसर उपिब्र् हैं ? [2]
v. ‘अंतराकष्ट्रीय योग कदिस’ पहिी बार िब मिाया गया ? इसिा श्रेय किसे जाता है ? [2]

Question 4 Answer the following according to the instructions given . [8]

3
निम्िनिखित प्रश्नों िे उत्तर निदे शािुसार निखिए |
i. निम्िनिखित में से किन्हीं दो शब्दों से भाििाचि संज्ञा बिाइए| [1]

(ि) बूिा (ि) र्बरािा (ग) चििा (र्) नचकित्सि


ii. निम्िनिखित में से किन्हीं दो शब्दों से विशेषण बिाइए| [1]

(ि) दशकि (ि) अकहं सा (ग) र्ि (र्) सम्माि


iii. निम्िनिखित में से किन्हीं दो शब्दों िे दो पयाकयिाची शब्द निखिए | [1]

(ि) मछिी (ि) पुत्री (ग) बफक (र्) िस्त्र

iv. निम्िनिखित में से किन्हीं दो शब्दों िे वििोम शब्द निखिए | [1]

(ि) मूि (ि) भाग्यिाि (ग) बंर्ि (र्) पुरस्िृ त


v. निम्िनिखित मुहािरों में से किसी एि िा िाक्य में प्रयोग िीखजए | [1]

(ि) छाती पर सााँप िोटिा (ि) दाि में िािा होिा


vi. निम्िनिखित प्रश्नों िे उत्तर निदे शािुसार निखिए :- [3]

ि बच्चे गीत गा रहे हैं | (िाक्य िो संकदग्र् भूतिाि में बदनिए )

ि आओ, ज़रा र्ूमें | (िाक्य िो भाििाच्य में बदनिए)

ग िह र्िी है पर उसमें अनभमाि िहीं है | (िाक्य िो नमश्र िाक्य में बदनिए )

SECTION B [40 MARKS]

साकहत्य सागर
Question 5 Read the extract given below and answer in Hindi the questions that [10]
follows :-
निम्िनिखित गद्ांश िो पकिए और उसिे िीचे नििे प्रश्नों िे उत्तर कहं दी में निखिए :-
एि मत से यह तय हो गया कि िि-प्रदे श में प्रजातंत्र िी स्थापिा हो गया कि िि-
प्रदे श में प्रजातंत्र िी स्थापिा हो। पशु-समाज में इस ‘क्राखन्तिारी’ पररितकि से हषक िी
िहर दौड़ गई कि सुि-समृवद्ध और सुरिा िा स्िणक-युग अब आया और िह आया |”
[ भेड़ें और भेकड़ए – हररशंिर परसाई ]
i. प्रजातंत्र किसे िहते हैं ? ’िि प्रदे श में प्रजातंत्र िी स्थापिा हो’ - से क्या तात्पयक है ? [2]
ii. ‘क्राखन्तिारी पररितकि’ िा क्या अथक है ? पशु-सामाज में हषक िा क्या िारण था ? [2]
iii. सुि-समृवद्ध और सुरिा िा स्िणक–युग शीघ्र आिे िािा है ,ऐसा किसे और क्यों िग रहा है ? [3]

िारण सकहत उत्तर निखिए |

4
iv. पाठ में भेड़ें और भेकड़ए किसिे प्रतीि हैं ? क्या जंगि में प्रजातंत्र आया ? [3]

विस्तार से निखिए |

Question 6 Read the extract given below and answer in Hindi the questions that [10]
follows :-
निम्िनिखित गद्ांश िो पकिए और उसिे िीचे नििे प्रश्नों िे उत्तर कहं दी में निखिए :-
”अब िह िया व्यंजि बिािे बैठी | हांड़ी में दे िा तो र्ी पाि भर से अनर्ि ि था ।
बड़े र्र िी बेटी किफ़ायत क्या जािे । उसिे सब र्ी मांस में डाि कदया |”
[ बड़े र्र िी बेटी – प्रेमचंद ]
i. किसिे, किसिे निए िया व्यंजि बिाया और क्यों ? [2]
ii. सारा र्ी मांस में डाििे िा क्या पररणाम हुआ ? विस्तार से निखिए । [2]
iii. प्रस्तुत िहािी िे शीषकि िी साथकिता स्पष्ट िीखजए । [3]
iv. िाि वबहारी द्वारा किए गए व्यिहार िी समीिा िरते हुए उसिा चररत्र नचत्रण िीखजए । [3]

Question 7 Read the extract given below and answer in Hindi the questions that [10]
follows :-
निम्िनिखित गद्ांश िो पकिए और उसिे िीचे नििे प्रश्नों िे उत्तर कहं दी में निखिए :-
”िह प्रायः अिेिा बैठा-बैठा शून्य मि से आिाश िी ओर तािा िरता | एि कदि उसिे
ऊपर आसमाि में पतंग उड़ती दे िी । ि जािे क्या सोचिर उसिा हृदय एिदम खिि
उठा, विश्वेश्वर िे पास जािर बोिा, ”िािा ! मुझे एि पतंग माँगा दो ।’’
[ िािी – नसयारामशरण गुप्त ]
i. यहााँ िक्ता िौि है , िह उदास क्यों है ? [2]
ii. उड़ती हुई पतंग िो दे ििर उसिा हृदय क्यों खिि उठा ? [2]
iii. िक्ता और विश्वेश्वर में क्या ररश्ता है ? िक्ता िा चररत्र नचत्रण िीखजए | [3]
iv. क्या विश्वेश्वर िे िक्ता िी मााँग िो पूरा किया ? विश्वेश्वर िा व्यिहार उनचत था या [3]

िहीं ? अपिे विचार स्पष्ट िीखजए ।

एिांिी संचय

Question 8 Read the extract given below and answer in Hindi the questions that [10]
follows :-
निम्िनिखित गद्ांश िो पकिए और उसिे िीचे नििे प्रश्नों िे उत्तर कहं दी में निखिए :-
”मुझे किसी िे बताया ति िहीं | यकद िोई नशिायत थी तो उसे नमटा दे िा चाकहए था ।

5
हल्िी-सी िरोंच भी, यकद उस पर तत्िाि दिाई ि िगा दी जाए तो बििर एि बड़ा र्ाि
बि जाती है और यही र्ाि िासूर हो जाता है , कफर िाि मरहम िगाओ ठीि िहीं
होता।”
[ ‘सूिी डािी’ – उपेन्द्रिाथ ’अश्ि’ ]
i. उक्त िथि िा सन्दभक बताइए । [2]
ii. िक्ता िा पररचय दीखजए । [2]
iii. नशिायत नमटािा क्यों जरुरी होता है , ि नमटािे पर क्या होता है ? [3]
iv. श्रोता िौि है , उसिे िक्ता िो किसिे बारे में क्या जाििारी दी ? [3]

Question 9 Read the extract given below and answer in Hindi the questions that [10]
follows :-
निम्िनिखित गद्ांश िो पकिए और उसिे िीचे नििे प्रश्नों िे उत्तर कहं दी में निखिए :-
”र्ाय मााँ, तुमिे उदय नसंह िे सामिे तो अपिे पुत्र चन्दि िो भी भुिा कदया । तुम्हारे
मातृत्ि में उदय नसंह ऐसे समाए हैं , जैसे िटार िो अपिे ह्रदय में रििे िे निए म्याि िे
अपिा हृदय िोििा िर कदया हो ।”
[ ‘दीपदाि’ – डॉ. रामिुमार िमाक ]
i. उपयुक्त
क िथि िे िक्ता तथा श्रोता िौि है ? [2]
iii. चन्दि िौि है और उसे िौि, किसनिए भुिा रहा है ? [2]
iii. गद्ांश िा सन्दभक बताते हुए र्ाय मााँ िे चररत्र िी विशेषताओं पर प्रिाश डानिए । [3]
iv. एिांिी िे शीषकि िी साथकिता स्पष्ट िीखजए । [3]

Question 10 Read the extract given below and answer in Hindi the questions that [10]
follows :-
निम्िनिखित गद्ांश िो पकिए और उसिे िीचे नििे प्रश्नों िे उत्तर कहं दी में निखिए :-
” रमेश गया है गौरी िो विदा िरािे । (ििाई पर बंर्ी र्ड़ी दे ििर) िुछ ही दे र में उसे
िेिर आता ही होगा | मेरी बेटी पहिा सािि यहााँ वबताएगी । तुम्हारी बहि िे सपिे
िभी पूरे िहीं होंगे और उसिे सपिों िे िूि िा दाग तुम्हारे हाथों और तुम्हारी मााँ िे
आाँचि पर होगा ! समझे? ”
[ ‘बहू िी विदा’ – वििोद रस्तोगी ]
i. प्रस्तुत िथि िे िक्ता और श्रोता िौि हैं ?रमेश और गौरी िा िक्ता से क्या संबंर् है ? [2]
ii. ‘बहू िी विदा’ एिांिी िा उद्दे श्य स्पष्ट िीखजए । [2]

6
iii. िक्ता क्या चाहता है ? उसिा सोचिा सही है या गित? इस पर अपिे विचार प्रिट [3]

िीखजए ।
iv. श्रोता िौि है ? उसिा चररत्र-नचत्रण िीखजए । [3]

*****

7
VIBGYOR HIGH
Sample Paper
History and Civics
Grade: IX Max. Marks: 80
Date: Time Allowed: 2 hours

INSTRUCTIONS:
 Answers to this paper must be written on the paper provided separately.
 You will not be allowed to write during the first 15 minutes.
 This time is to be spent in reading the question paper.
 The time given at the head of this paper is the time allowed for writing
the answers.
 The intended marks for the questions or parts of questions are given
alongside the questions.
 This question paper consists of 4 pages.

PART 1 (30 marks)


Attempt all questions from this part
Question 1 [10]
(a) State the significance of January 26. [1]
(b) What is meant by a “Welfare State”? [1]
(c) By which Act did the government of India grant dual citizenship to [1]
all the Persons of Indian Origin?
(d) Mention one Fundamental Right granted to citizens of India. [1]
(e) Explain one feature of Single Citizenship. [1]
(f) When is a mid-term election held? [1]
(g) What is a By-Election? [1]
(h) Mention any one function of the Gram Panchayat. [1]
(i) Name any one function of the Municipal Committee. [1]
(j) State any one important function of the Zila Parishad. [1]

1
Question 2 [20]
(a) Name the four Vedas. [2]
(b) Name any two gods of the Early Vedic Period. [2]
(c) Write two causes for the rise of Jainism and Buddhism. [2]
(d) Name any two doctrines of Jainism. [2]
(e) What is meant by the term ‘Sangam Age’? [2]
(f) Name the village assemblies that existed during Chola dynasty. [2]
(g) Which is the best example of Chola temples and to whom is it [2]
dedicated?
(h) Name the three sections into which the chapters of Tirukkural are [2]
categorized and its author.
(i) What is known as the ‘gopuram’ and ‘mandapa’? [2]
(j) Who built the Red fort? Mention any one distinctive architectural [2]
features of the Red fort.

PART II (50 marks)


Section A (CIVICS)
Attempt any two questions from this Section

Question 3 With reference to the Constitution of India and the Local Self [10]
Government answer the following:
(a) What is meant by the term Constitution? When was the [3]
Constitution adopted and when did it come into force?
(b) Enumerate some of the functions of the Panchayat Samiti. [3]

(c) State any four functions of the Municipal Corporation. [4]

Question 4 With reference to the salient features of the Indian [10]


Constitution answer the following questions:
(a) Explain the basic features of Universal Adult Franchise. [3]
(b) List any three Fundamental Duties enlisted in the Constitution of [3]
India.
(c) What are Directive Principles of State Policy and in which part of [4]
the Constitution are they incorporated in?

2
Question 5 With reference to the Elections and the Election [10]
Commission, answer the following:
(a) Explain the term ‘General Elections’. [3]
(b) State the composition of the Election Commission. [3]
(c) Distinguish between Direct Election and Indirect Elections. [4]

SECTION B
Attempt any three questions from this section
Question 6 The knowledge of the Harappan civilisation forms the basis [10]

of our understanding of Human civilisation in general, and


Indian culture in particular. In the light of this statement,
answer the following:

(a) Describe the significance of the Seals as a source of information. [3]


(b) How were the granaries built and what was their importance? [3]
(c) Elaborate on the importance of the Great Bath. [4]

Question 7 With reference to the Delhi Sultanate era, answer the [10]
following:
(a) What measures did Alauddin Khilji take to increase the revenue? [3]
(b) What were the military reforms introduced by Alauddin Khilji? [3]
(c) Why did Muhammad bin Tughlaq increase the taxation in the [4]
Doab and what were its consequences?

Question 8 With Reference to the Age of the Guptas, answer the [10]
following questions:
(a) Explain how the Allahabad Pillar Inscription is a source of [3]
Information about the Gupta Age?
(b) Mention the contribution of Kalidasa to the Gupta period. [3]
(c) What contribution did Aryabhatta make in the field of science, [4]
astronomy and mathematics?

3
Question 9 With reference to the Modern age in Europe, answer the [10]
following:

(i) (ii) (iii)


(a) Identify the given pictures. [3]
(b) How did the invention of Printing Press bring in a new awakening [3]
in Europe?
(c) How did the first personality given in the picture contribute in the [3]
field of Science?

Question 10 With reference to the Modern age in Europe , answer the [10]
following:
(a) What is meant by Industrial Revolution? Define capitalism and [3]
socialism.
(b) Mention any three church practices that caused dissatisfaction [3]
among the people.
(c) Explain Reformation and Counter reformation ? [4]

*****

4
VIBGYOR HIGH
SAMPLE PAPER
MATHEMATICS
Grade: IX Max. Marks: 80
Date: Time Allowed: 2½ hours

INSTRUCTIONS: -
 Answers to this paper must be written on the paper provided separately.
 You will not be allowed to write during the first 15 minutes.
 This time is to be spent in reading the question paper.
 The time given at the head of this paper is the time allowed for writing
the answers.
 Attempt all questions from Section A and any four question from Section B
 The intended marks for the questions or parts of questions are given
alongside the questions.
 All working, including rough work, must be clearly shown and must be
done on the same sheet as the rest of the answer. Omission of
essential working will result in the loss of marks.
 Geometrical figures to be constructed wherever applicable.
 For geometry, figures are to be copied to the answer script.
 This question paper contains 5 printed pages.

SECTION A (40 marks)


(Attempt all questions)

Question 1 a) If 2x – 3y = 10 and xy = 16 ; find the value of 8x3 – 27y3 [3]


b) Evaluate: [3]
5 1
54  125  3   25 

2

c) Calculate the difference between the compound interest and [4]


the simple interest on ₹ 12,000 at 10 % p.a for 1 years.

1
Question 2 a) If 3x + 1 = 9x-3, find the value of 21 + x [3]
b) A chord of length 24 cm is at 5 cm from the centre of the circle. [3]
Find the length of the chord of the same circle which is at 12 cm
from the centre.
c) Factorize: [4]
4(2x-3y)2 -8x + 12y – 3

Question 3 a) Use the information in the given figure to prove: [3]


(i) AB = FE (ii) BD = CF

b) AD is drawn perpendicular to the base of an equilateral triangle [3]


ABC. Given BC = 10cm, find the length of AD, correct to 2
places of decimal.
c) The angles of a quadrilateral are in the ratio 3 : 4 : 5 : 6. [4]
Show that the quadrilateral is a trapezium.

Question 4 a) Given: 4 cot A = 3, find sec2 A – cot2A [3]


b) Construct a frequency polygon without histogram from the [3]
following frequency table:
Class Interval Frequency
0 – 12 12
12 – 24 16
24 – 36 15
36 – 48 10
48 - 60 18

c) A metal wire when bent in the form of an equilateral triangle of [4]


largest area, encloses an area of 484 3 cm2 . If the same wire
is bent into the form of a circle of largest area, find the area of
this circle..
Section B (40 marks)

2
Attempt any 4 questions

Question 5 a) If x - y + z = 5 and x2 + y2 + z2 =49, find the value of [3]


zx – xy – yz.
b) Solve by substitution method: 3x – 2y = -7 and 5x -3y = 13 [3]
c) If x = 1 - 2 , find the value of (x - )3 [4]

Question 6 a) Given A = 600 and B = 300 , prove that : [3]


sin (A + B) = sin A cos B + cos A sin B
b) The mean weight of 30 students of a class is 60.2 kg. Two [3]
students of weight 50 kg and 67 kg left the class. Find the mean
weight of the remaining students.
c) A certain sum of money at compound interest amounts to [4]
₹ 6,600 in 1 year and to ₹ 7,986 in 3 years. Find the sum
and the rate percent.

Question 7 a) Find the median for the following data: [3]


50, 59, 27, 78, 54, 72, 58, 57, 31, 65, 42, 60,22, 37, 40 and 28
b) Construct a cumulative frequency distribution table for the given [3]
frequency table and use the table given below to find:
(i) The class boundaries of the fifth class.
(ii) The actual class limits of the sixth class.
Class Interval Frequency
30 – 34 7
35 - 39 10
40 – 44 12
45 – 49 13
50 – 54 8
55 – 59 4

c) Factorise: [4]
i) a2 - b2 – (a + b)2
ii) 3p(a2 +b2) – 5q (a2 +b2) – 4r (a2 +b2)

3
Question 8 a) Find the value of x and y if: [3]
(5x – 3y , y – 3x) = (4,- 4)
b) D and F are mid points of sides AB and AC of a triangle ABC. A [3]
line through F and parallel to AB meets BC at E. Prove that
BDFE is a parallelogram.
c) Solve the given equations graphically: [4]
x + 2y = 4 ; 3x – 2y = 4

Question 9 a) Prove that the points A(1,-3), B(-3,0), and C(4,1) are the [4]
vertices of an isosceles right angled triangle.
b) [6]
i) If tan A + cot A = 5. Find the value of tan2 A + cot2 A. [2]
ii) Construct the frequency distribution table from the following [4]
cumulative frequency table:
Ages No. of students
Below 4 0
Below 7 85
Below 10 140
Below 13 243
Below 16 300
(i) State the number of students in the age group 10 – 13.
(ii) State the age-group which has the least number of
students.

Question 10 a) Draw the graph of the equation 3y + 5 = 0 [3]


b) Solve : + =2 and + = 22 [3]

c) In triangle ABC, AB = AC and BD is perpendicular to AC. Prove [4]


that: BD2 – CD2 = 2 CD x AD.

Question 11 a) The sum of the digits of a two digit number is 7. If the digits are [4]
reversed, the new number increased by 3, equals 4 times the
original number. Find the original number.

4
b) [6]
i) A small indoor green house (herbarium) is made entirely of [4]
glass panes including the base, held together with tape. It is
30cm long, 25cm wide and 25cm in height.
What is the area of the glass used?
How much tape is needed for all the 12 edges?
ii) If tan 2 = cot ( + 6º ), find the angle [2]

*****

5
VIBGYOR HIGH
Sample Paper
MASS MEDIA & COMMUNICATION

Grade: IX Max. Marks: 100


Date: Time Allowed : 2 hours

INSTRUCTIONS:-
 Answers to this paper must be written on the paper provided separately.
 You will not be allowed to write during the first 15 minutes.
 This time is to be spent in reading the question paper.
 The time given at the head of this paper is the time allowed for writing the
answers.
 The intended marks for the questions or parts of questions are given
alongside the questions.
 Attempt ALL questions from Section A and any four questions from
Section B.
______________________________________________________________________
Section A (40 Marks)
Attempt all questions from this Section.
Q.1
a) List the advantages of written communication. [2]
b) What are the differences between formal and informal communication? [2]
c) What do you understand with medium in the communication process? [2]
Explain with example/s.
d) Why it is important to understand audience in media? [2]
e) Explain the internet as mass media. [2]

Q.2
a) What do you understand by copyright and plagiarism? [2]
b) How to avoid clutter while developing/designing any print media [2]
creative collateral.
c) What is a Berliner? Give an example. [2]
d) What is the difference between AM and FM? [2]
e) Explain public radio broadcasting. [2]

1
Q.3
a) Define advertisement. Explain the purpose of advertising. [2]
b) Why should advertisement have objectives? [2]
c) What is the difference between advertising using traditional media and
advertising through new age media? Explain with examples. [2]
d) What does consistency mean in advertising? How can consistency
in advertising campaign help in increases revenue? [2]
e) Mention names of four advertising tycoons/gurus. [2]

Q.4
a) What is a web browser and what is a search engine? Explain with [2]
examples.
b) What do you understand by online broadcast/webcast? Explain with an [2]
example.
c) What is electronic mailing? How can it be used as a channel for digital [2]
marketing?
d) Explain gaming and how can gaming be interactive. [2]
e) What Is File Transfer Protocol (FTP)? [2]

Section B (60 Marks)


Attempt any four questions from this Section
Q.5
a) How would you describe effective communication? [5]
b) Context is one of the basic components of human communication. [5]
Discuss.
c) Explain the process of communication and its elements. [5]

Q.6
a) Discuss any five types of media with examples. [5]
b) Television is an audio-visual medium. Discuss this as you explain the
functions of television. [5]
c) Describe radio equipment. [5]

2
Q.7
a) Print revolution resulted in the reading mania. Comment. [5]
b) Write a short note on magazines as print media. [5]
c) Why do companies print brochures? [5]

Q.8
a) What are the different types of ethical issues in advertising? [5]
b) Simplicity and creativity are two principles of advertising. Is it possible
to have an ad campaign that is creative yet simple and the vice versa?
Explain with examples. [5]
c) What are the functions of advertising? Explain any five with examples. [5]

Q.9
a) What do you understand by Blog, Blogging and Blogger? Can a
blogger be an influencer? How does influencer aid in marketing?
Explain. [5]
b) Social network, in computers, is an online community of individuals
who exchange messages, share information, and, in some cases,
cooperate on joint activities. Discuss. [5]
c) Write a short note on online banking. [5]

Q.10
Please read the below short case and answer the questions
below.
Amazon's Business Model

Often touted as the largest online retailer in the world, Amazon


operates a business model with many moving parts. First and foremost,
the company sells goods directly. A percentage of products are offered
to buyers through Amazon's online storefront with a small markup, and
inventory is kept in the company's large network of warehouses. Most
consumers visit the company's site assuming its products are less
expensive and readily available for purchase and shipping.

In addition to direct sales, Amazon provides a platform for other


retailers to sell products to buyers. Products sold through Amazon's
partner retailers are often less common items or those with a higher
purchase price, allowing Amazon to avoid holding slow-moving
inventory that could dilute profit. While Amazon does not assess a fee
for its retailer partners to list items for sale, the company does retain a

3
portion of the sales price as commission.

Amazon also maintains a subscription-based business model through


its Amazon Prime service, as well as a small electronics product line.
Under a Prime account, customers pay an annual fee to secure free
two-day or same-day shipping on eligible items and have access to
streaming media, such as digital music or movies. Amazon also
generates revenue from selling its e-reader, the Kindle, and the e-book
and mobile application purchases offered to Kindle owners.

Source: Invesopedia.com

a) Describe the B2B and B2C models of communication in Amazon from


advertising point of view. Support your answer with diagram/s. [5]
b) What is digital marketing? If Amazon is to be used as a channel for
digital marketing, how do you think you can best use this platform for
advertising? [5]
c) What are the advantages and disadvantages of e-commerce? [5]

*****

4
VIBGYOR HIGH
Sample Paper

PHYSICAL EDUCATION
Grade: IX Max. Marks: 100
Date : Time Allowed: 2 Hours

Instructions:-

● Answers to this paper must be written on the paper provided separately.

● You will not be allowed to write during the first 15 min.


● This time is to be spent in reading the question paper.
● The time given at the head of this paper is the time allowed for writing the
answers.
● The intended marks for question or parts of question are given in brackets [ ]
● Attempt all questions from section A and any two questions from section B.

SECTION A (50 marks)


Attempt all the questions from this Section

Question 1
a) Explain flexion and extension. [2]

b) Define respiration. [2]

c) Write any three functions of blood. [3]

d) Define following terms: [3]

1) Heart rate

2) Stroke volume

3) Cardiac output

1
Question 2

a) Write short note on hamstring group of muscles. [2]

b) Define Vital capacity and Oxygen Debt. [2]

c) Explain the double circulation of blood in body. [3]

d) Write any six benefits of exercise on respiratory system. [3]

Question 3

a) “There is no I in team.” Explain it in the light of teamwork. [2]

b) Distinguish between voluntary and involuntary muscles. [2]

c) Briefly explain diffusion of gases in alveoli. [3]

d) Name the type of joints which are found in the following : [3]

1) Shoulder and Hip

2) Elbow and Knee

3) Fingers and Thumbs

Question 4
a) Distinguish between isometric and isotonic muscle contraction. [2]

b) Write any four benefits of exercise on muscular system. [2]

c) How does hinge joint work? How is the hinge joint held in position? Name [3]
movement possible at this joint.

d) Differentiate between RBC and WBC on the basis of following characteristics: [3]

1) Life span

2) Shape

3) Functions

2
Question 5
a) Write down the functions of following parts of respiratory system. [2]

1) Nasal cavity

2) Larynx

b) Distinguish between games and sports. (Any two points) [2]

c) Write any 3 characteristic features of cardiac muscle. [3]

d) Give exact location of the following muscles in human body. [3]

1) Deltoid

2) Pectorals

3) Gastrocnemius

SECTION B (50 marks)

Attempt any two questions from this Section

You must attempt one question on each of the two games of your choice

FOOTBALL
Question 6
a) Draw a specified diagram of football ground giving all its dimensions. [8]

b) Answer the following questions:


1. Write the duration of match, duration of extra time and radius of corner arc. [9]
2. When is a goal kick awarded? Explain its procedure.
3. State any six direct free kick offences committed by a player in football.

c) Solve the following questions. [8]


1. Write a short note on AIFF.
2. What is abbreviation and meaning of GLT?
3. What is meant by sudden death?
4. Explain any four offences for which a red card is shown to a player.

3
Question 7
a) Explain the following terms in football: [8]
1. Added time
2. Tackling
3. Through pass
4. Corner kick

b) Answer the following questions: [9]


1. State any three duties of an assistant referee.
2. Mention any three offences for which a yellow card is shown to a player.
3. What is the procedure of taking the corner kick?

c) Answer the following questions: [8]


1. What is free kick?
2. What do you mean by Ball out of play?
3. What is advantage in football?
4. Mention any four skills in football.

BASKETBALL

Question 8
a) Explain the following terms in basketball: [8]
1. Charging
2. Man to man defence
3. Rebounding
4. Backcourt

b) Answer the following questions: [9]


1. What is a throw-in? Explain its procedure.
2. List any three occasions when ball becomes dead in basketball match.

3. What is the duration of the game and extra time in basketball?

4
c) Answer the following questions: [8]
1. What do you mean by Double dribble?
2. What is Eight second rule?
3. What is meant by Five foul by player?
4. Write a short note on FIBA.

Question 9
a) Draw the diagram of Basketball court and list its dimensions. [8]

b) Answer the following questions: [9]


1. What is Substitution? Write the procedure of Substitution.
2. Mention any three duties of the timekeeper.
3. What is Three second rule?

c) State the following terms: [8]


1. Jump stop
2. Dead ball
3. Fast break
4. Technical foul

CRICKET
Question 10
a) Answer the following questions: [8]
1. What do you mean by ‘Follow on’?
2. What is Ball tampering?
3. What is full form of IPL and ODI?
4. What do you mean by ‘Danger area’?

b) Answer the following questions: [9]


1. Name any six cricket fielding positions outside the 30 yards circle.
2. Mention any three instances when umpire gives call for no ball.
3. What is 30 yard circle in the game?

5
c) State the following terms: [8]
1. Beamer
2. Leg byes.
3. Free hit
4. Bowl out

Question 11

a) Answer the following questions: [8]

1. What is full form of ICC and BCCI?

2. List down any four batting techniques played by batsman.

3. In what occasions game can be suspended?

4. What do you mean by Declaration?

b) Answer the following questions: [9]


1. Mention any three interruptions that are classified as intervals.
2. Write down any six ways of getting out in cricket.
3. What is meant by extra runs? Give two examples of extra runs.

c) State the following terms: [8]


1. Off spin.
2. Power play
3. Yorker
4. Overthrow

*****

You might also like